GI exammaster
A 23-year-old Caucasian woman presents with weakness, fatigue, and occasional palpitations starting 6 months ago. She took a multivitamin supplement with transient improvement. In the last 3 months, blisters appeared on her elbows, which were associated with intense itching. Her only medication is an implantable contraceptive inserted 3 years ago. There is no relevant past medical history, and her family history is also unremarkable. Vital signs are BP 120 / 80 mmHg, HR 94 bpm, RR 18 rpm temperature 36.3° C (97.2° F). On physical examination, she is alert and oriented, with no dyspnea or cyanosis. Examination of the skin reveals bullous lesions in her elbows. The remainder of the examination shows no abnormalities. A complete blood count shows Hemoglobin 12 g/dL, Hematocrit 31%, Mean Corpuscular Volume (MCV) 78 fl, Mean Corpuscular Hemoglobin Concentration (MCHC) 29 pg/L, Leukocytes 7.400 /mm3 w/normal differential, Platelets 218,000 /mm3. A skin biopsy reveals a neutrophilic infiltrate, with fibrin and microblisters, most dense at the tips of the dermal papillae and containing IgA on immunofluorescence staining. Question What finding is most probable on subsequent workup? Answer Choices 1 Positive transglutaminase antibodies 2 Positive ASCA antibodies 3 A warm spot on scintigraphy with 99mTc-labelled red cells 4 Positive stool guaiac test 5 Reduced serum erythropoietin
1 Positive transglutaminase antibodies Dermatitis herpetiformis produces erythematous, pruriginous bullous lesions on the extensor surfaces of the elbows, knees, scalp, neck, and buttocks. These lesions, sometimes called skin celiac disease, are found in 10% of patients with celiac disease, and are highly specific for it; nearly all patients with dermatitis herpetiformis have histologic evidence of celiac disease on small-bowel biopsy. However, in many cases the enteropathy is subclinical. Histopathologic examination of the bullae reveals deposits of anti-gliadin IgA antibodies in the basal membrane of the epidermis. In most cases, the skin lesions involute after the patient is put on a gluten-free diet. However, this can take several months. In patients with severe pruritus, the mainstay of treatment is dapsone, which is highly effective in relieving this symptom. Second-line options include sulfasalazine and sulfapyridine. Celiac disease can present with isolated nutrient deficiencies (e.g. iron, folate, B-complex vitamins), and therefore could explain this patient's iron deficiency anemia. As transglutaminase antibodies are found in 70% of cases of celiac disease, there is a high probability that this assay will be positive in this patient, whether or not the anemia is caused by celiac disease. The more florid presentation of celiac disease - with overt steatorrhea, weight loss, and malnutrition - is becoming less common in the United States. Therefore, the physician must be aware of the possibility of celiac disease in patients with milder symptoms, such as occasional diarrhea with bulky stools, bloating, or more rarely, isolated micronutrient deficiencies. Anti-Saccharomyces (ASCA) antibodies are associated with Crohn's disease (60-70% of patients are positive). Lesions in these patients include oral aphthous ulcers, pyoderma gangrenosum, and rheumatoid nodules, as well as signs of non-cutaneous disease (e.g. enterocutaneous fistulas, malnutrition, micronutrient deficiencies). A 99mTc-red cell scintigraphy could be positive because occult gastrointestinal bleeding is still a possibility. Causes of a negative iron balance include: 1) Deficient intake, which is most prevalent in poor countries. 2) Pregnancy. 3) Chronic bleeding (gynecologic, gastrointestinal, hematuria). 4) Iron deposition in tissues (e.g. cardiac hemosiderosis) The deficiency in hemoglobin synthesis in the bone marrow tends to cause a compensatory rise in erythropoietin levels, which produces erythroid hyperplasia in the bone marrow. Other findings include absence of sideroblasts and stainable iron in the bone marrow's reticular network. However, bone marrow biopsy has been largely abandoned for diagnosis of iron-deficiency anemia because of interobserver variability and the availability of less invasive tests of iron status, particularly serum ferritin and total iron binding capacity (TIBC).
A 55-year-old man presents with a 12-hour history of severe epigastric abdominal pain that radiates into his back, nausea, vomiting, and chills. His pertinent physical examination findings include oral temperature of 102°F and epigastric tenderness upon palpation. He is admitted to the hospital for management of his condition. Question What criteria would increase the likelihood of a severe attack associated with a high incidence of short and long term morbidity and mortality? Answer Choices 1 White blood cell (WBC) count of 10,000cells/mm3 upon admission 2 A 14% decrease of his hematocrit at 48 hours after admission 3 Serum glucose of 130 mg/dL upon admission 4 Aspartate transaminase of 240 U/dL upon admission 5 Serum calcium of 10 mg/dL 48 hours after admission
2 A 14% decrease of his hematocrit at 48 hours after admissionA patient who is admitted to the hospital for treatment of acute pancreatitis is assessed using the Ranson criteria to determine the severity of their disease, which in turn helps determine prognosis. About 70 - 80% of cases of acute pancreatitis are considered mild and result in virtually no morbidity or mortality. The remainder are severe attacks and have a 10 - 30% mortality rate. Fast identification of the severe cases is helpful to reduce the morbidity and mortality for each patient. There are 11 Ranson criteria, 5 of which are determined upon admission, and 6 at 48 hours after admission. Patients who have 2 or fewer of the criteria have minimal mortality. Patients with 3 - 5 of the criteria have about a 10 - 20% chance of mortality. Patients with 5 or more of the criteria have at least a 50% mortality rate. A patient with at least a 10% decrease in their hematocrit at 48 hours after admission meets one of the Ranson criteria, so the correct answer is a 14% decrease in hematocrit 48 hours after admission. WBC count of 10,000 cells/mm3 is not one of the Ranson criteria. A WBC count of >16,000 cells/mm3 would meet one of the Ranson criteria. Serum glucose of 130 mg/dL upon admission is also not one of the Ranson criteria. The patient's serum glucose would have to be at least 200 mg/dL to count as one of the Ranson criteria and increase the severity of their disease. Aspartate transaminase of 240 U/dL upon admission is not one of the Ranson criteria either. The patient's aspartate transaminase would have to be at least 250 U/dL to count as a Ranson criteria and increase the severity of their disease. A serum calcium of 10 mg/dL 48 hours after admission is not one of the Ranson criteria. If the patient's serum calcium is under 8 mg/dL 58 hours after admission, this would count as one of the Ranson criteria.
A 38-year-old man presents with sudden onset of acute upper abdominal pain since the previous night associated with nausea, several episodes of vomiting, and weakness. The pain is mostly in the epigastric region with constant, severe, and steady radiation to the back. He also has a low grade fever since this morning without any chills. He denies diarrhea or dysuria. His past history is significant for hypertension, for which he takes amlodipine 10 mg daily and enalapril 5 mg daily. He has smoked half a pack of cigarettes daily for the past 12 years. He initially denies drinking alcohol except on the weekends occasionally, but on further and repeated questioning he says he drinks 3-4 beers daily and had been drinking continuously for the last 2 days with his friends while watching sports on TV. Family history is unremarkable. On examination he has a temperature of 100.6° F, pulse 106/minute, BP 150/92 mm Hg, and respiratory rate is 20/minute. There is no pallor, icterus, cyanosis, or lymphadenopathy. Mucus membranes are dry, and skin is somewhat clammy. Lungs are clear, and heart sounds normal except for sinus tachycardia. Abdominal exam reveals diffuse tenderness in the epigastric and right as well as left upper quadrants. There is some distension and mild guarding in the upper abdomen. Bowel sounds are hypoactive, but there is no ascites or hepatosplenomegaly. Rectal exam is normal. Labs reveal Hb 15g%, WBC 14,500/uL, platelets 400,000/uL, AST 42 U/L, ALT 36 U/L, AP 26 U/L, amylase 3600 U/L, lipase 546 U/L, BUN 25 mg/dL, creatinine, 1.5 mg/dL, bilirubin 1.2 mg/dL, and random blood sugar 110 mg/dL. Question Which of the following is one of the predictors of acute pancreatic necrosis if present at diagnosis along with 2 other factors? Answer Choices 1 Age over 50 years 2 WBC count more than 16,000/uL 3 Blood glucose over 180mg/dL 4 Serum LDH over 300 U/L 5 AST more than 200 U/L
2 WBC count more than 16,000/uL Explanation This patient is suffering from acute alcoholic pancreatitis. The 2 most common causes of acute pancreatitis are gallstones and alcohol. Other causes include hypertriglyceridemia, hypercalcemia, abdominal trauma, ERCP, and drugs like valproic acid, azathioprine, mercaptopurine, didanosine, thiazides, tetracyclines etc. Assessment of severity is done by either Ranson's criteria on admission and at 48 hours or by the Acute Physiology and Chronic Health (APACHE) II scoring system. Ranson's criteria include: (1) Age more that 55 years (2) WBC count more than 16,000/Ul (3) Blood glucose more than 200mg/dL (4) Serum LDH over 350 U/L (5) AST over 250 U/L. 3 or more criteria on admission predict a complicated course with possibility of pancreatic necrosis. At 48 hours development of any of the following indicates a worsening prognosis: (1) Hematocrit drop of more than 10% (2) BUN rise greater than 5 mg/dL (3) Arterial PO2 less than 60 mm hg (4) Serum calcium less than 8 mg/dL (5) Base deficit over 4 meq/L (6) Estimated fluid sequestration of more than 6 L. An elevated C reactive protein at 48 hours suggests the development of pancreatic necrosis. A high amylase and lipase are suggestive of acute pancreatitis. Leukocytosis is usually present. Other than the labs ordered above, imaging may also be done. Plain abdominal X-ray may show radio-opaque gallstones, the sentinel loop sign (localized ileus of a small segment of small intestine, usually in the left upper quadrant), and the colon cut-off sign (lack of air in the colon in the area of the inflamed pancreas immediately preceded by a gas filled segment of transverse colon). There may be a reactive pleural effusion with atelectasis in the lower lobes of the lungs. Ultrasound is non specific and may show gallstones. CT scan of the abdomen will show the inflamed pancreas and detect complications like necrosis or pseudocyst formation. Since Ranson's criteria and APACHE scoring systems are cumbersome and time consuming, a CT Severity Index (CTSI) has recently become popular. It uses a grading system based on unenhanced CT of the pancreas and a necrosis score based on contrast enhanced CT of the pancreas. CTSI is the unenhanced score plus the necrosis score, the maximum of which can be 10 and more than 6 indicates severe disease. MRI and MRCP are being increasingly used where available for diagnosis and management of pancreatitis. Treatment involves bowel rest, aggressive hydration, pain control, and bed rest in a hospital. Electrolyte imbalances should be corrected, especially calcium since saponification may lower the level. Broad spectrum antibiotics are not routinely recommended, but in patients with impending necrotizing pancreatitis, antibiotics have shown to decrease mortality. Nutritional support is of utmost importance. For mild to moderate cases, IV hydration and gentle advancement of oral feeding is recommended, where as in severe cases total parenteral nutrition may be needed. Alcohol abstinence will be needed to prevent further episodes. Complications include prerenal azotemia, acute tubular necrosis, shock, pancreatic necrosis, pseudocyst formation, ARDS, and pancreatic abscess. Intra-abdominal hemorrhage may cause ecchymoses around the umbilicus (Cullen's sign) or in the flanks (Grey-Turner sign), though these are rare and not specific for pancreatitis. Prognosis is good for mild cases who abstain from drinking. Recurrences are common in alcoholics. Prognosis is poor for severe necrotizing pancreatitis, especially with multi-organ involvement.
A 62-year-old woman has an unexplained weight loss. She has a vague, non-descriptive pain in her stomach accompanied by diarrhea. When her son sees that she has become yellow, he insists that she seek medical attention. Numerous tests are done, including endoscopic retrograde pancreatography, and she is told that she has pancreatic cancer. The tests reveal that the pancreatic cancer is located at the head of the pancreas. Question What percentage of pancreatic cancers are located at the head of the pancreas? Answer Choices 1 1-10 2 20-30 3 45-55 4 70-80 5 90-99
70-80 About 70-80% of pancreatic cancers are located at the head of the pancreas. Despite the fact that pancreatic tumors that arise in the head of the pancreas may present with obstructive symptoms, the tumor still is usually advanced by the time of diagnosis. Other pancreatic cancers are located in the body and tail of the pancreas. Tumors that arise in the tail of the pancreas are even more insidious.
A 55-year-old man presents with a 12-hour history of severe epigastric abdominal pain that radiates into his back, nausea, vomiting, and chills. His pertinent physical examination findings include oral temperature of 102°F and epigastric tenderness upon palpation. He is admitted to the hospital for management of his condition. Question What criteria would increase the likelihood of a severe attack associated with a high incidence of short and long term morbidity and mortality? Answer Choices 1 White blood cell (WBC) count of 10,000cells/mm3 upon admission 2 A 14% decrease of his hematocrit at 48 hours after admission 3 Serum glucose of 130 mg/dL upon admission 4 Aspartate transaminase of 240 U/dL upon admission 5 Serum calcium of 10 mg/dL 48 hours after admission
A 14% decrease of his hematocrit at 48 hours after admission is the correct response. A patient who is admitted to the hospital for treatment of acute pancreatitis is assessed using the Ranson criteria to determine the severity of their disease, which in turn helps determine prognosis. About 70 - 80% of cases of acute pancreatitis are considered mild and result in virtually no morbidity or mortality. The remainder are severe attacks and have a 10 - 30% mortality rate. Fast identification of the severe cases is helpful to reduce the morbidity and mortality for each patient. There are 11 Ranson criteria, 5 of which are determined upon admission, and 6 at 48 hours after admission. Patients who have 2 or fewer of the criteria have minimal mortality. Patients with 3 - 5 of the criteria have about a 10 - 20% chance of mortality. Patients with 5 or more of the criteria have at least a 50% mortality rate. A patient with at least a 10% decrease in their hematocrit at 48 hours after admission meets one of the Ranson criteria, so the correct answer is a 14% decrease in hematocrit 48 hours after admission. WBC count of 10,000 cells/mm3 is not one of the Ranson criteria. A WBC count of >16,000 cells/mm3 would meet one of the Ranson criteria. Serum glucose of 130 mg/dL upon admission is also not one of the Ranson criteria. The patient's serum glucose would have to be at least 200 mg/dL to count as one of the Ranson criteria and increase the severity of their disease. Aspartate transaminase of 240 U/dL upon admission is not one of the Ranson criteria either. The patient's aspartate transaminase would have to be at least 250 U/dL to count as a Ranson criteria and increase the severity of their disease. A serum calcium of 10 mg/dL 48 hours after admission is not one of the Ranson criteria. If the patient's serum calcium is under 8 mg/dL 58 hours after admission, this would count as one of the Ranson criteria.
A 55-year-old woman presents with poor appetite and nausea. She has vomited 2 times over the past week and lost 4 pounds in the past month. Past medical history is significant for 20 years of alcoholism, hypertension, and diabetes for the past 5 years. She takes no medications; she is not involved in any therapy for her alcoholism. She has been drinking 2 pints of beer almost every day for the past 20 years, and consumes greater quantities on weekends. Her vitals include a heart rate of 102 BPM, blood pressure of 140/100 mm Hg, respiratory rate of 20/min, and a temperature of 99.8°F. Physical exam reveals hepatomegaly. A liver biopsy reveals ballooning degeneration, spotty necrosis, and polymorphonuclear infiltration. Question What lab finding would be characteristic of this patient's condition? Answer Choices 1 AST/ALT >2 2 Reduced γ-Glutamyl transpeptidase 3 Reduced alkaline phosphatase 4 Reduced prothrombin time 5 Microcytic hypochromic anemia
AST/ALT>2 The patient's history of alcoholism, physical findings, and liver biopsy results point towards a diagnosis of alcoholic liver disease (ALD), where typically the AST/ALT >2 (the ratio of serum aspartate aminotransferase to serum alanine aminotransferase). Patients with increased alcohol consumption may not have a ratio >2, and it may be more of an indication of advanced alcoholic liver disease. ALD is a result of the consumption of large quantities of alcohol over a long duration of time. The threshold for men is 60 - 80 g/d for > 10 years. Initially there is a fatty infiltration of the liver, which gradually progresses to alcoholic hepatitis. Alcoholic hepatitis is characterized by hepatocyte injury, as revealed by the biopsy findings in this patient. Lab findings include a moderately elevated AST, ALT, γ-Glutamyl transpeptidase, and alkaline phosphatase. Both AST and ALT are elevated by 2 - 7 fold, and AST is greater than ALT level (AST/ALT >2). Bilirubin is also elevated. Prothrombin time is >5 seconds (normal is 11 - 15 seconds), and macrocytic anemia may also be observed.
A 60-year-old man presents with severe abdominal pain that started 10 hours ago. It is increasing in severity and is colicky in nature. The patient has not had a bowel movement for 3 days. In the last 2 days, he vomited up what he ate 4 times. He looks tired and dehydrated. On examination, increased bowel sounds were noticed. There is also a mass in the right inguinal area. The patient said he the mass has been present for 10 years but disappears when he lies on his back. Question What is the investigation of choice to confirm the diagnosis? Answer Choices 1 Abdominal X-Ray Erect film 2 Abdominal Ultrasound 3 CBC 4 Colonoscopy 5 Upper GIT endoscopy
Abdominal X-ray Erect film. Intestinal obstruction occurs when bowel movement is encountering an obstacle in the passage through the bowel. Actually this can be caused by any mass, adhesion, or any other means of mechanical obstruction. The symptoms of obstruction vary according to the site of obstruction, but generally there is increased bowel movement proximal to obstruction with the reflected increase in bowel sounds. There may be vomiting. There may be no bowel movement, from absolute constipation to feces and flatus. Here the patient has inguinal hernia which is recently irreducible and obstructed. The best diagnostic procedure is erect X-ray film which shows multiple fluid levels and gives confirmation of the diagnosis. The treatment is exploratory surgery, and there is no place for any conservative treatment without surgery. Paralytic ileus is a type of intestinal obstruction that usually follows general anesthesia and is characterized by silent abdomen with no bowel sounds, usually treated by bowel rest and proper hydration. Colonoscopy here is contraindicated. Upper GIT endoscopy is generally used to detect pathology in upper GIT, not in the case of intestinal obstruction. Abdominal ultrasound is of value in diagnosis of gall bladder disease and also urinary bladder pathology. CBC is generally used to detect leukocytosis in acute appendicitis or inflammatory reactions.
A 41-year-old man presents with right upper quadrant pain. His pain began gradually following a meal, but it has now become constant. He notes that he has had previous episodes of similar pain, but it has never been quite this severe. The pain radiates to his right shoulder and is worsened with inspiration. He has experienced nausea and vomiting, and he notes feeling chilled. Examination reveals an overweight man in moderate distress. He develops rigors during the physical exam. He has scleral icterus, and his skin has a yellow hue. Heart and lungs are clear. His abdomen is soft and non-distended with positive Murphy's sign. Vital signs reveal BP 109/62, pulse 112, respirations 18, and temperature 102.3 degrees F. Laboratory studies include: Component Value: WBC-15.66 k/uL Hemoglobin-15.5 g/dL Hematocrit-43.8 % Platelets-128 k/uL Albumin-3.4 g/dL Total Bilirubin-6.3 mg/dL Conjugated Bilirubin-4.5 mg/dL Alkaline Phosphatase-114 U/L AST-97 U/L ALT-132 U/L Total Protein-5.6 g/dL Glucose-104 mg/dL BUN-11 mg/dL Creatinine-1.05 mg/dL Sodium-139 mmol/L Potassium-4.3 mmol/L Chloride-102 mmol/L CO2-27 mmol/L Calcium-8.5 mg/dL Amylase- 36 U/L Lipase- 23 U/L Question What is the most appropriate initial diagnostic study for this patient? Answer Choices 1 Abdominal X-ray 2 Abdominal MRI 3 Abdominal ultrasound 4 HIDA scan 5 ERCP
Abdominal u/s Abdominal ultrasound is correct. The above patient is suffering from right upper quadrant pain, jaundice, and fever, which are all part of the Charcot triad associated with cholangitis. Cholangitis is defined as infection of the bile ducts. It typically occurs when the bile ducts are obstructed, leading to infection. Given his history of postprandial right upper quadrant pain, the patient likely has cholelithiasis. His laboratory data is suggestive of biliary obstruction. The best initial testing for gallstones is abdominal ultrasound. Abdominal X-ray is incorrect. Plain abdominal X-ray may demonstrate radiopaque gallstones, but it is not the best test to demonstrate cholelithiasis because it may miss radiolucent stones. Abdominal MRI is incorrect. While abdominal MRI may show cholelithiasis, it is not the preferred initial study for suspected cholangitis. HIDA scan is incorrect. Hepatobiliary iminodiacetic acid (HIDA) scan may be used to demonstrate ductal obstruction, but it is typically done following abdominal ultrasound. ERCP is incorrect. If the patient in this case has obstructing stones, he may require endoscopic retrograde cholangiopancreatography (ERCP) for stone removal. However, ERCP would not be the initial study performed.
A 44-year-old pre-menopausal Caucasian woman with a BMI of 36 presents with persistent upper right quadrant abdominal pain that radiates to the back. It has gotten so bad that she has difficulty eating any food and needs to force herself to eat. She has nausea with some episodes of vomiting. She denies bulimia but admits to anorexia. On exam, patient has a positive Murphy's sign and tenderness to palpation on the epigastric and upper right quadrant area. Patient has a slightly elevated temperature. The physician assistant is awaiting labs and imaging. Question What is the most likely diagnosis? Answer Choices 1 Acute cholecystitis 2 Chronic cholecystitis 3 Appendicitis 4 Cholangitis 5 Gastritis
Acute cholecystitis Patients with acute cholecystitis present with abdominal pain accompanied by nausea and vomiting. Usually the patient consumes a large fatty meal 1 hour before the onset of symptoms; however, the pain is persistent. This usually occurs due to cholelithasis blocking the cystic duct. Patients commonly affected by cholelithasis are those who meet the four Fs: fat, female, forty, and fertile, which describes the patient above. Chronic cholecystitis is usually an intermittent type of pain and a mild inflammation of the gallbladder. This can turn into an acute cholecystitis. Appendicitis is usually right lower quadrant pain that presents as a diffuse abdominal pain that localizes to the right lower quadrant. The patient would not have a positive Murphy sign, but would have McBurney's point, obturator sign, and psoas sign. The patient would have a fever as well. Cholangitis is defined as a jaundice, fever, and right upper quadrant abdominal pain known as Charcot's triad. Gastritis is inflammation of the stomach, which is localized in the epigastric area. This can be caused by numerous reasons but is most commonly secondary to infectious or autoimmune causes.
A 32-year-old man comes to the doctor's room complaining of nausea, vomiting, dark urine, and yellow eyes for the last 3 - 4 days. He lives alone, and he admits to having more than 1 sexual partner but denies any intravenous drug use. He usually buys his food from different fast food places. He doesn't remember any contact with anybody who had hepatitis. On examination, he is slightly dehydrated, he has jaundice, tender hepatomegaly, splenomegaly, and the rest of the examination is normal. The doctor suspects viral hepatitis, so he sends bloods to the laboratory to check his hepatitis markers. The results are as follows: Anti-HAV- of IgM type is positive Anti-HAV- of Ig G type is negative HBsAg is positive HBsAb is negative HBcAb of IgG type is positive HBeAg is negative HBeAb is positive Question What is the most likely diagnosis? Answer Choices 1 Resolved hepatitis B and acute hepatitis A infection 2 Acute hepatitis A and B infections 3 Acute hepatitis B and past hepatitis A infection 4 Past hepatitis A and resolved hepatitis B infection 5 Acute hepatitis A infection and inactive carrier hepatitis B state
Acute hepatitis A infection and inactive carrier hepatitis B state
A 57-year-old woman with a 40 pack-year history of smoking and a long history of Type 2 Diabetes presents with fatigue, middle abdominal pain, and loss of appetite. She indicates that these bouts have been coming and going for the last few months and she finally decided to get checked out. She indicates that she has also lost considerable weight. On physical examination, she appears mildly jaundiced. You perform a CT scan that reveals a mass in the pancreas. Question What is the most likely diagnosis? Answer Choices 1 Gastrinoma 2 Adenocarcinoma 3 Teratoma 4 Insulinoma 5 Pancreatic sarcoma
Adenocarcinoma The clinical picture is suggestive of a chronic pancreatitis that has developed into pancreatic cancer. 90% of all pancreatic cancers are adenocarcinomas. The rest are adenosquamous, anaplastic, and acinar cell carcinomas. Virtually all pancreatic carcinomas (99%) originate in duct cells. A gastrinoma is a tumor that secretes the hormone gastrin. It is commonly found in the duodenum and is associated with Zollinger-Ellison syndrome. Symptoms include diarrhea, epigastric pain, hematemesis, and difficulty eating. Teratoma is a neoplasm consisting of all 3 germ layers. Insulinoma is a tumor of the pancreas associated with the beta-cells. Sarcomas are neoplasms of connective tissue. Pancreatic cancers are of epithelial cell origin.
A 55-year-old woman is broughtin by her daughter, who claims that her mother has lost 4 pounds over the last month, has poor appetite, and complains of nausea. She has vomited twice over the past 1 week. History is significant for alcoholism for the past 15 years and hypertension and diabetes for the past 5 years, for which she takes no treatment. She has been drinking 2 pints of beer almost every day for the past 20 years and greater quantities on weekends. Vitals: BP 140/100 mm Hg, HR 102/min, RR 20/min, and Temp 99.8 degrees F. Physical exam reveals hepatomegaly. A liver biopsy reveals ballooning degeneration, spotty necrosis, and polymorphonuclear infiltration. Question What is the most likely diagnosis? Answer Choices 1 Alcoholic fatty liver 2 Alcoholic hepatitis 3 Alcoholic cirrhosis 4 Alcoholic siderosis 5 Hepatocellular carcinoma
Alcoholic Hepatitis This patient most likely has alcoholic hepatitis, as her biopsy reveals ballooning degeneration, spotty necrosis, and polymorphonuclear infiltration. Other findings that may be seen are fibrosis of the perivenular and perisinusoidal space of Disse. Mallory bodies may be present. Alcoholic Liver Disease (ALD) begins with a fatty infiltration of the liver (fatty liver), which gradually progresses to alcoholic hepatitis. Alcoholic hepatitis is characterized by hepatocyte injury, as revealed by the biopsy findings in this patient. Lab findings include a moderately elevated AST, ALT, γ-Glutamyl transpeptidase, and alkaline phosphatase. Both AST and ALT are elevated by 2 - 7 fold, and AST is typically greater than ALT level (AST/ALT >1). Bilirubin is also elevated. Prothrombin time is >5 seconds, and macrocytic anemia may also be observed. Alcoholic cirrhosis refers to the irreversible injury caused to the liver parenchyma by excessive alcohol intake. Extensive fibrosis and formation of regenerative nodules is seen. Alcoholic siderosis is characterized by iron deposits in Kupffer cells in alcoholics. Alcoholic cirrhosis can progress to hepatocellular carcinoma, which is the primary malignant tumor of the liver.
A 4-day-old infant is seen for a routine newborn check; he has yellow skin from his face down to his chest. He is a full-term baby, had a normal vaginal delivery, weighed 3.180 kg at birth, and 3.100 kg at discharge. He is breastfeeding well, and stooling and voiding regularly. He is well-hydrated and active. Laboratory results reveal an indirect bilirubin of 12.2 mg/dL, and a direct bilirubin of 1.0 mg/dL. Question What is the physiologic mechanism behind this jaundice? Answer Choices 1 An elevation of direct bilirubin due to decreased bilirubin production 2 The increased production of red blood cells 3 The increased conjugation of bilirubin by the liver 4 An elevation of indirect bilirubin due to increased bilirubin production 5 The decreased production of red blood cells
An elevation of indirect bilirubin due to increased bilirubin production The correct response is an elevation of indirect bilirubin due to increased bilirubin production. Unconjugated indirect bilirubin enters the liver and is excreted from the hepatic cells in a water-soluble form (conjugated and direct) into the biliary system. The direct bilirubin leaves the body in the form of stool. The most common cause of indirect hyperbilirubinemia is physiologic jaundice, which peaks at 2 - 4 days of life. Physiologic jaundice is due to increased bilirubin production after the breakdown of red blood cells, in conjunction with limited conjugation of bilirubin by the immature liver. Physiologic jaundice can be worse in babies that are breastfed, due to a higher level of dehydration as the mother waits for her milk to come in.
A 42-year-old woman IV drug user presents with vague symptoms of fatigue, aches, pains, and nausea. Recently, she appeared jaundiced for a few days; however, she no longer does. Preliminary blood tests indicate the patient does not have hepatitis B. However, the tests could be wrong, as she may be in the window period. If she were in the window period, what would be the only evidence of hepatitis B infection? Answer Choices 1 Anti-HBc 2 HBsAg 3 HBcAg 4 HBeAg 5 Anti-HBs
Anti-HBc Hepatitis B is transmitted by exposure to blood or blood products. If she shared needles as an IV drug user, she has put herself at risk of infection with hepatitis B. The window is the period in which HBsAg (HBV surface antigen) is not detectable and anti-HBs (antibody to HBV surface antigen) has yet to appear. During the window period of a hepatitis B infection, the only marker present would be IgM anti-HBc (antibody to HBV core antigen). Because of this, the window period is also referred to as the core window. This IgM antibody is present in the early convalescence period (2 - 16 weeks after infection). It is an expensive test and is ordered only when there is very high suspicion. Positivity of this antibody, along with HbsAg, indicates acute infection. PCR test for genome identification is the other test useful during the window period. HBsAg (HBV surface antigen) is the 1st serological marker to appear in hepatitis B infection. After HBsAg is cleared from the serum, the patient is in the window period. HBeAg (HBV 'e' antigen) can be seen with acute hepatitis. It is a marker for infectivity. HBcAg is the HBV core antigen. Anti-HBs (antibody to HBV surface antigen) is seen at the end of an infection. The appearance of anti-HBs marks the end of the window period. The presence of anti-HBs gives a patient immunity.
A 32-year-old IV drug user presents with fatigue, joint pain, and nausea. She initially thinks she has the flu until she becomes visibly jaundiced. She sees her doctor and blood work is done. She is diagnosed with hepatitis B infection. She recovers without complications. On a follow up visit, she asks her doctor if she can get hepatitis B again or if she is immune. Question What would confer immunity from further infections of hepatitis B? Answer Choices 1 Anti-HBs 2 Anti-HBc 3 HBsAg 4 HBcAg 5 HBeAg
Anti-Hbs Hepatitis B is transmitted by exposure to blood or blood products. If this patient shared needles when using drugs, she put herself at risk of infection with hepatitis B. Anti-HBs (antibody to HBV surface antigen) is seen at the end of an infection. The appearance of anti-HBs marks the end of the window period. The presence of anti-HBs gives a patient immunity. HBcAg is the HBV core antigen. HBsAg (HBV surface antigen) is the first serological marker to appear in hepatitis B infection. After HBsAg is cleared from the serum, the patient is in the window period. The window is the period in which HBsAg (HBV surface antigen) is not detectable and anti-HBs (antibody to HBV surface antigen) is yet to appear. During the window period of a hepatitis B infection, the only marker present would be anti-HBc (antibody to HBV core antigen). Because of this, the window period is also referred to as the core window. HBeAg (HBV "e" antigen) can be seen with acute hepatitis. It is a marker for infectivity.
You are evaluating a 59-year-old male with history of heavy alcohol use for the past 5 years who appears acutely ill. Which of the following findings is most likely to be an indicator of increased mortality in this patient with severe alcoholic hepatitis? A fatty liver B INR of 7 C AST > ALT by a factor of 2:1, but less than 300 units/L D serum bilirubin 1.8mg/dl E folic acid deficiency
B INR of 7 The correct answer is choice (B). A significantly elevated PT/INR as well as bilirubin > 10 mg/dl are indicators of severe alcoholic hepatitis and increased mortality requiring hospitalization. Choices (A), (C), and (E) are all common findings in alcoholic hepatitis. Mild elevation of serum bilirubin, choice (D), is also common.
A 4-day-old male infant is seen for a routine newborn check; he has yellow skin from his face down to his chest. He is a full-term infant; vaginal delivery was normal. He weighed 3.180 kg at birth, and 3.100 kg at discharge. He is breastfeeding well; he is also stooling and voiding regularly. He is well-hydrated and active. Laboratory results reveal a direct bilirubin of 3.4 mg/dL, and an indirect bilirubin of 12.3 mg/dL. Question What is a possible pathologic cause of the jaundice? Answer Choices 1 Breast milk 2 Birth trauma 3 Breastfeeding 4 TPN cholestasis 5 Biliary atresia
Biliary atresia The correct response is biliary atresia. Breastfeeding contributing to dehydration and intensified physiologic jaundice, breast milk jaundice, and birth trauma causing cephalohematoma with increased hemolysis are all possible causes of indirect hyperbilirubinemia. Since the infant is 4 days old, his indirect bilirubin level is not so high as to require treatment or concern at this time. Any direct bilirubin >2.0 mg/dL is abnormal. If it is persistent or increasing it is considered pathologic; it must be evaluated immediately. TPN cholestasis and biliary atresia can both contribute to direct hyperbilirubinemia. TPN cholestasis can occur when an infant has received hyperalimentation for an extended period of time, so it would not occur in a 4-day-old infant. Cholestasis is impaired excretion of the conjugated bilirubin in bile. The most common hepatic cause of jaundice of direct hyperbilirubinemia in infancy is biliary atresia. Ultrasound should be performed to identify blockages or anatomical anomalies.
You are evaluating a 67-year-old male with known cirrhosis of the liver secondary to alcoholic liver disease, although he has been sober for the past year. He is brought in to the emergency department by his daughter, who notes that for the past few days he has seemed to be more confused. On examination you note the patient to be mildly confused but alert to person and place. He has noticeable asterixis. He is not currently taking any medications and his blood alcohol level is undetectable. What is the treatment of choice in this case based on your physical examination findings? A amoxicillin B prednisone C lactulose D folic acid E thiamine
C Lactulose The correct answer is (C). This patient most likely has hepatic encephalopathy due to end-stage liver disease. Asterixis indicates an increase in serum ammonia. The treatment of choice is lactulose. Both folic acid and thiamine are used in the treatment of alcoholic liver disease, but do not treat elevated ammonia levels. Antibiotics may be used secondarily in patients nonresponsive to lactulose, but amoxicillin is not preferred. Prednisone is not a treatment for hepatic encephalopathy.
A 63-year-old white male is seen in the ambulatory outpatient clinic with complaints of midepigastric pain, weight loss, and jaundice. On examination, he is jaundiced and his sclerae are icteric. On palpation of the abdomen, you find a distended nontender gallbladder. Which of the following is the most likely diagnosis? A Gastric carcinoma B Chronic pancreatitis C Pancreatic carcinoma D Choledocholithiasis
C Pancreatic Cancer Pancreatic carcinoma presents with weight loss, jaundice, and midepigastric pain. A palpable, nontender gallbladder (Courvoisier sign) is more often associated with a pancreatic malignancy than cholelithiasis, especially if the tumor is in the head of the pancreas. In acute cholecystitis, the obstruction in the cystic duct is associated with inflammation, resulting in a tender gallbladder on palpation of the right upper quadrant (Murphy sign); obstruction of the common bile duct in choledocholithiasis will result in jaundice but not weight loss. Gastric carcinoma will present with midepigastric pain and weight loss but not jaundice or a palpable gallbladder. Midepigastric pain is the most common symptom seen in chronic pancreatitis, and weight loss may be seen in association with malabsorption secondary to exocrine insufficiency.
A 43-year-old obese man presents for a health maintenance visit. On physical exam, it is noted that his waist circumference is 106 cm and blood pressure is 148/92 mm Hg. Which of the following fasting laboratory levels would suggest a diagnosis of metabolic syndrome (syndrome X) in this patient? A HDL of 45 mg/dL B LDL of 180 mg/dL C triglyceride of 190 mg/dL D glucose of 100 mg/dL
C trig of 190 mg/dL Metabolic syndrome is found in approximately 25% of Americans. It is defined as three or more of the following findings: waist circumference of greater than 102 cm in men or greater than 88 cm in women; serum triglyceride level of at least 150 mg/dL, HDL level of less than 40 mg/dL in men or less than 50 mg/dL in women; blood pressure of at least 130/85 mm Hg; and serum glucose level of at least 110 mg/dL.
Tumor marker for pancreatic cancer
CA 19-9
A 65-year-old man presents with abdominal pain, vomiting, and weakness. On examination, he is cachectic, pale, and jaundiced. A walnut-sized mass is palpable in the right hypochondrium. What is the most likely diagnosis? Answer Choices 1 Carcinoma of head of pancreas 2 Acute or chronic cholecystitis 3 Fibrolipoma 4 Hydatid cyst of the liver 5 Primary biliary cirrhosis
Carcinoma of the head of the pancreas Carcinoma of the head of pancreas is a common 'duodenal C-loop cancer'. Seen mostly in men, it is usually a ductal adenocarcinoma. It causes obstructive jaundice, weight loss, itching, and upper abdominal pain. There may be diabetes, bleeding problems, malabsorption syndromes, and LFT showing obstructive jaundice. This carcinoma has a better prognosis than cancer of the body and tail of the pancreas when resection is possible. Courvoisier's law states that when the gall bladder is palpable (but not tender) in the presence of jaundice, the cause is usually cancer. Cholecystitis is a diagnosis of 'female, fat, fertile, forty, and flatulent'. It does occur in men. Jaundice may or may not be present. Vomiting usually brings relief to the upper abdominal pain. Muscle guarding and rebound tenderness is present. A fibrolipoma over the right hypochondrium may be a coincidental finding, but would not be the cause of the overall clinical features of a malignancy. A hydatid cyst results from an E. Granulosus infection, and cysts may form in the liver, lung and other organs. It may become palpable in the right hypochondrium and cause pressure symptoms. Migrant workers and immigrants from endemic areas may carry this infection. Primary biliary cirrhosis is an autoimmune disease that destroys the liver structure and system; it is mostly seen in women over the age of 40. There is itching, xanthomata, and hepatosplenomegaly.
A 48-year-old man presents with right upper quadrant pain. His pain developed after eating fried chicken at a cookout, and it is now continuous. He has experienced previous episodes of less severe right upper quadrant pain; they resolved spontaneously. The pain radiates to his right shoulder; it is accompanied by nausea, vomiting, and chills. On physical examination, the patient is noted to have scleral icterus and jaundice. Rigors are present. The abdomen is slightly distended, with positive Murphy's sign. Vital signs include a BP of 120/85 mm Hg, pulse 117, respirations 18, and temperature 102.5 degrees F. CBC reveals mild leukocytosis, elevations of serum aminotransferases and bilirubin, as well as normal lipase. Question What is the likely diagnosis in this patient? Answer Choices 1 Cholelithiasis 2 Cholangitis 3 Pancreatitis 4 Cirrhosis 5 Hepatocellular carcinoma
Cholangitis Cholangitis is the correct response. The patient has right upper quadrant pain, jaundice and fever, which are all part of the Charcot triad associated with cholangitis. Cholangitis is an infection of the common bile duct that typically occurs when the duct becomes obstructed. The patient reports previous episodes of right upper quadrant pain; they were likely caused by gallstones (cholelithiasis). The patient probably has a stone obstructing the bile duct, causing cholangitis. Cholelithiasis is an incorrect response. The patient likely does have gallstones. However, gallstones do not always lead to cholangitis. It is only when a stone becomes obstructed that cholangitis may develop. Therefore, while cholelithiasis is likely contributing to the patient's diagnosis, cholangitis is a more appropriate diagnosis. Acute pancreatitis is an incorrect response. The pain associated with pancreatitis is usually epigastric, with radiation into the back. Pancreatitis may be associated with jaundice if the cause is gallstone pancreatitis. However, if the patient had pancreatitis, his lipase would be elevated. Cirrhosis and hepatocellular carcinoma are both incorrect responses. While these conditions may be associated with jaundice, the presence of Charcot's triad strongly suggests cholangitis.
A 45-year-old man presents with a 1-week history of high-grade fever, abdominal pain, occasional rigors, loss of appetite, malaise, nausea, and vomiting. He was all right about a week ago when he had low-grade fever for the first 2 days. Fever was high grade during the last 3 - 4 days, with occasional rigors accompanied by abdominal pain predominantly in right upper abdomen. Pain is moderate in intensity, continuous without any radiation, and is not relieved by acetaminophen (Tylenol), which he is taking on his own. Examination reveals tender hepatomegaly. No other organs are palpable. Abdomen is slightly distended, but bowel sounds are normal. Vital parameters: temp 40° C, pulse 108 per minute, blood pressure 136/90mmHg, and respiration rate 20 per minute. Patient appears sick and toxic. The rest of the physical examination is as following: Neck: Supple. No stridor. Lymphatics: No lymph node groups were palpable. Lungs: Decreased breath sounds at the base of the right lower lung. Cardiac: Normal S1 and S2. No extra heart sounds. No heave. Back: Negative Groin: Unremarkable Genitalia: Unremarkable Neurological: Normal cranial nerves. Normal motor and sensory examination. Ultrasonography reveals multiple liver abscesses in both lobes of the liver, which is confirmed by similar findings in CT scan. Question What is the most common cause of the patient's pyogenic liver abscess? Answer Choices 1 Pylephlebitis 2 Cholangitis 3 Endocarditis 4 Sub-phrenic abscess 5 Pyelonephritis
Cholangitis Cholangitis is the most common cause of pyogenic liver abscess. Biliary pathologies are the most common predisposing cause of pyogenic liver abscess and accounts for 60% of the cases. The etiological classification of bacterial abscess of the liver (pyogenic liver abscess) is made on the basis of the route by which infection reaches the liver. There are 5 mechanisms: Pylephlebitis is the inflammation of the portal vein or any of its branches. Portal bacteremia [via portal vein] from an intra-abdominal site, such as diverticulitis, suppurative appendicitis, and infected carcinoma of the colon, especially after resection, ulcerative colitis, inflamed hemorrhoids, typhoid, and paratyphoid fever. Along the bile ducts, causing ascending cholangitis in a biliary tract partially or completely obstructed by stone, tumor, or stricture. Systemic bacteremia originating in a distant location, with organisms reaching the liver via the hepatic artery; causes enumerated can be septicemia and pyemia, endocarditis, pyelonephritis, and infection of the hydatid cyst. Direct extension from an adjacent infection outside the biliary tract, such as from a sub-diaphragmatic abscess, from an empyema thoracis, and from trauma, either penetrating (with direct implantation of bacteria into the liver) or blunt, causing a hematoma that becomes secondarily infected. From the umbilicus along the umbilical vein of the newborn and along the paraumbilical veins. A cause is usually obvious, but sometimes the abscess is unexplained. Although most abscesses are single, multiple (usually microscopic) abscesses are common with systemic bacteremia or complete biliary tract obstruction. Prompt antibiotic treatment, with agents such as meropenem, imipenem, and cefuroxime, followed by percutaneous drainage, form the essential components of treatment. Biliary tract diseases account for 60% of the pyogenic liver abscess cases. Systemic bacteremia accounts for 15% of the cases. 24% of the pyogenic liver abscesses can be attributed to pylephlebitis. Direct extension from an adjacent infection like sub-phrenic abscess accounts for 4% of the cases.
A 56-year-old man presents with abdominal pain, indigestion, weight loss, nausea, and vomiting, and gray colored stools for the past month. His past medical history is positive for alcoholism. Lab results demonstrate an elevated serum amylase and lipase, decreased trypsinogen, and a positive fecal fat test. What is the most likely diagnosis? Answer Choices 1 Acute pancreatitis 2 Chronic pancreatitis 3 Pancreatic carcinoma 4 Pacreatic abscess 5 Insulinoma
Chronic Pancreatitis Pancreatitisis an inflammation or infection of the pancreas. Chronic pancreatitis, which is the correct response, is caused by alcohol abuse, hemochromatosis (a condition of excess iron in the blood), and other unknown factors. Inflammation and fibrosis cause the destruction of functioning glandular tissue in the pancreas. This results in an inability to properly digest fat caused due to a lack of pancreatic enzymes. The production of insulin is also affected. Symptoms include abdominal pain (mainly in the upper abdomen), nausea, vomiting, weight loss, and fatty stools. Additional symptoms may include swelling (overall), stools (clay colored), and abdominal indigestion. Tests should include serum lipase (may be elevated), serum amylase (may be elevated), serum trypsinogen (may be low), and fecal fat test (shows fatty stools). Abdominal ultrasound and CT may show an enlarged pancreas. Treatment of chronic pancreatitis includes reducing pancreas stimulation, alleviating fat indigestion, reducing pain, and treating diabetes. A reduced-fat diet, vitamin supplementation, no alcohol or caffeine, and regulation of blood sugar levels are indicated in the treatment. The chief causes of acute pancreatitis in adults are gallstones, other biliary disease, and alcohol use. Viral infection (mumps, Coxsackie B, mycoplasma pneumonia, and Campylobacter), injury, pancreatic or common bile duct surgical procedures, and certain medications (especially estrogens, corticosteroids, thiazide diuretics, acetaminophen, tetracycline) are other causes. After the triggering event, the process continues with autodigestion that causes swelling, hemorrhage, and damage to the blood vessels. An attack may last for 48 hours. Symptoms include abdominal pain (mainly located in the upper abdomen) nausea, vomiting, weakness, sweating, anxiety, fever, clammy skin, and mild jaundice. General examination may show a low blood pressure and a heart rate above 90. Most cases resolve within a week with supportive measures, such as analgesics and fluid replacement. However, some cases can be life threatening. Pancreatic abscess occurs in 5 to 10% of people with acute pancreatitis. An abscess may be caused by inadequate drainage of a pancreatic pseudocyst, which is a complication associated with pancreatitis. Symptoms include fever, chills, abdominal pain, and abdominal mass. Physical exam will show signs of pancreatitis, and tests should include an abdominal CT and ultrasound. Treatment will include laparotomy with drainage and possible resection of dead tissue. Pancreatic cancer is the 4th most common cancer causing death in the U.S. The disease is more common in men, especially those between 60 and 70 years. The cause is unknown; however the incidence is greater in smokers. A high fat diet and chemical exposures may increase the risk. Symptoms include weight loss, abdominal pain, loss of appetite, jaundice, nausea, weakness, fatigue, vomiting, diarrhea, indigestion, back pain, stools (clay colored), pallor, and depression. Tests should include a pancreatic biopsy, an abdominal CT scan, and abdominal ultrasound. Only 20% of the tumors are operable at the time of diagnosis. Palliation is generally the treatment, along with chemotherapy and radiation. Insulinomas are generally benign tumors of the insulin-secreting cells of the pancreas, which secrete excess amounts of insulin. Risk factors include a prior history of multiple endocrine neoplasia Type I (MEN I). Symptoms include sweating, tremor, rapid heart rate, anxiety, hunger, dizziness, headache, clouding of vision, confusion, behavioral changes, convulsions, and loss of consciousness. Surgery is the treatment of choice to remove the tumor. If the tumor is not found during surgery, diazoxide may be given. A diuretic is always given with this medication to avoid retaining too much salt.
A 42-year-old man presents with severe abdominal pain in the mid-epigastric region. The pain began 3 days ago with an intensity of 10/10, concomitant nausea, and vomiting (on 2 occasions). The patient denies any significant past medical history. BP is 86/58, and pulse is 112; temp is 101.5° F. There is rebound tenderness in the midepigastric area; bowel sounds are absent, and there is a positive Cullen's sign. The rest of the physical exam is within normal limits. What are the 2 most common causes of acute pancreatitis? Answer Choices 1 Hyperlipidemia and hypercholesterolemia 2 Pancreatic cancer and chemotherapeutic drugs 3 Gallstones and alcohol abuse 4 Abdominal trauma and postoperative period 5 Peptic ulcer disease and cystic fibrosis
Gallstones and alcohol abuse More than 70% of cases of acute pancreatitis are due to alcohol abuse or gallstones. Less common causes include postoperative pancreatitis, abdominal trauma, hyperlipidemia, certain drugs, hypercalcemia, peptic ulcer disease, uremia, cystic fibrosis, and viral infections. Complications such as adult respiratory distress syndrome, hemorrhagic pancreatitis, pancreatic abscess, pancreatic pseudocyst, and pancreatic ascites account for the 10% mortality rate associated with acute pancreatitis. Hemorrhagic pancreatitis can lead to retroperitoneal hemorrhage and widespread tissue necrosis, and it may require surgical intervention or peritoneal lavage. This retroperitoneal bleeding can be responsible for the discoloration or hematoma in the periumbilical area (Cullen's sign).
A 13-year-old boy presents with a long history of intermittent jaundice without other signs and symptoms. Yesterday he took several paracetamol tablets, and the headache and jaundice appeared again. His parents have a consanguineous marriage, and nobody in the family has similar symptoms. His physical examination today is within normal limits, except for mild scleral icterus. The initial laboratory examinations show: RBC 4.5mill/mm3; WBC 6000/mm3; total bilirubin 2.2 mg/dl; indirect bilirubin 2.0 mg/dl; direct bili 0.2 mg/dl; liver enzymes, serum copper, and all other parameters are normal; hepatitis B (-); and the full set of his autoimmune markers (-) and Coombs test (-). Blood smear is normal. Ultrasonography of liver, gallbladder, pancreas, spleen, and for both kidneys are normal. His urine appears very yellow. Question What is the most likely the diagnosis? Answer Choices 1 Gilbert's syndrome 2 Intravascular hemolysis 3 Chronic viral hepatitis 4 Crigler-Najjar syndrome 5 Budd-Chiari syndrome
Gilbert's disease Your patient most probably has Gilbert's syndrome (GS), also known as Gilbert-Meulengracht syndrome. It is a relatively common genetic disease found in up to 5% - 10% of the population and generally does not need special treatment. Inherited non-haemolytic hyperbilirubinemic conditions include Dubin-Johnson, Rotor, and GB syndromes, and all are important differential diagnoses indicating benign disease that requires no immediate treatment. GB can be diagnosed by clinical presentation, biochemistry, and genotyping, and is significant because of the presence of the disposition towards drug-associated toxicity. A major characteristic is jaundice, caused by elevated levels of unconjugated bilirubin in the bloodstream. The cause of this hyperbilirubinemia is the reduced activity of the enzyme glucuronyltransferase, which conjugates both bilirubin and some lipophilic molecules, including drugs. Intravascular hemolysis, with resulting hemoglobinemia, hemoglobinuria, and bilirubinemia, will show fragments of the red blood cells ("schistocytes") and sometimes spherocytes in peripheral blood smear, reticulocytosis, elevated unconjugated bilirubin that may lead to jaundice, elevated lactate dehydrogenase (LDH) in the blood, and decreased haptoglobin levels. If the direct Coombs test is positive, hemolysis is caused by an immune process. Hemosiderin in the urine indicates chronic intravascular hemolysis. There is also urobilinogen in the urine. Viral hepatitis with jaundice will have elevated liver function tests (AST and ALT elevated out of proportion to alkaline phosphatase, usually with hyperbilirubinemia), and viral serologic testing will be positive. Crigler-Najjar syndrome is a rare inherited form of non-hemolytic jaundice, which results in high levels of unconjugated bilirubin and often leads to brain damage in infancy. Budd-Chiari syndrome is caused by occlusion of the hepatic veins. It presents with the classical triad of abdominal pain, ascites, and hepatomegaly. The syndrome can be fulminant, acute, chronic, or asymptomatic.
A 13-year-old boy presents with a long history of intermittent jaundice without other signs and symptoms. Yesterday he took several paracetamol tablets, and the headache and jaundice appeared again. His parents have a consanguineous marriage, and nobody in the family has similar symptoms. His physical examination today is within normal limits, except for mild scleral icterus. The initial laboratory examinations show: RBC 4.5mill/mm3; WBC 6000/mm3; total bilirubin 2.2 mg/dl; indirect bilirubin 2.0 mg/dl; direct bili 0.2 mg/dl; liver enzymes, serum copper, and all other parameters are normal; hepatitis B (-); and the full set of his autoimmune markers (-) and Coombs test (-). Blood smear is normal. Ultrasonography of liver, gallbladder, pancreas, spleen, and for both kidneys are normal. His urine appears very yellow. Question What is the most likely the diagnosis? Answer Choices 1 Gilbert's syndrome 2 Intravascular hemolysis 3 Chronic viral hepatitis 4 Crigler-Najjar syndrome 5 Budd-Chiari syndrome
Gilbert's syndrome Your patient most probably has Gilbert's syndrome (GS), also known as Gilbert-Meulengracht syndrome. It is a relatively common genetic disease found in up to 5% - 10% of the population and generally does not need special treatment. Inherited non-haemolytic hyperbilirubinemic conditions include Dubin-Johnson, Rotor, and GB syndromes, and all are important differential diagnoses indicating benign disease that requires no immediate treatment. GB can be diagnosed by clinical presentation, biochemistry, and genotyping, and is significant because of the presence of the disposition towards drug-associated toxicity. A major characteristic is jaundice, caused by elevated levels of unconjugated bilirubin in the bloodstream. The cause of this hyperbilirubinemia is the reduced activity of the enzyme glucuronyltransferase, which conjugates both bilirubin and some lipophilic molecules, including drugs. Intravascular hemolysis, with resulting hemoglobinemia, hemoglobinuria, and bilirubinemia, will show fragments of the red blood cells ("schistocytes") and sometimes spherocytes in peripheral blood smear, reticulocytosis, elevated unconjugated bilirubin that may lead to jaundice, elevated lactate dehydrogenase (LDH) in the blood, and decreased haptoglobin levels. If the direct Coombs test is positive, hemolysis is caused by an immune process. Hemosiderin in the urine indicates chronic intravascular hemolysis. There is also urobilinogen in the urine. Viral hepatitis with jaundice will have elevated liver function tests (AST and ALT elevated out of proportion to alkaline phosphatase, usually with hyperbilirubinemia), and viral serologic testing will be positive. Crigler-Najjar syndrome is a rare inherited form of non-hemolytic jaundice, which results in high levels of unconjugated bilirubin and often leads to brain damage in infancy. Budd-Chiari syndrome is caused by occlusion of the hepatic veins. It presents with the classical triad of abdominal pain, ascites, and hepatomegaly. The syndrome can be fulminant, acute, chronic, or asymptomatic.
A 42-year-old man presents with acute abdominal pain. He describes the pain as constant and severe. Upon further questioning, he affirms that the pain radiates to his back. He leans forward in attempt to alleviate the pain. On physical examination, his temperature is 101 F and his pulse is 94. His abdomen is tender, but there is no guarding or rigidity. There is discoloration in the flanks and discoloration around his umbilicus. His lab results are as follows: TEST RESULTS REFERENCE RANGE Potassium 4.2 mEq/L 3.5 - 5.0 mEq/L Calcium 8.1 mg/100 mL 8.7 - 10.5 mg/100 mL Amylase 378 U/L 25 - 125 U/L Lipase 630U/L 30 - 210 U/L Question The discoloration of the flanks in this patient is indicative of what sign? Answer Choices 1 Grey Turner's sign 2 Cullen's sign 3 Lisker's sign 4 Kehr's sign 5 Blumberg's sign
Grey Turners Grey Turner's sign is ecchymosis of the flanks that can sometimes be seen with acute pancreatitis. This patient has signs and symptoms of acute pancreatitis,which include non-colicky abdominal pain that radiates to the back and fever. An elevated amylase, an elevated serum lipase,and hypocalcemia are all also consistent with pancreatitis. The most frequent underlying cause of pancreatitis is alcoholism or biliary disease. Cullen's sign is ecchymosis around the umbilicus that can sometimes be seen with acute pancreatitis. Lisker's sign is tibial bone tenderness that can sometimes be elicited with deep vein thrombosis. Kehr's sign is pain in the left shoulder secondary to splenic rupture. Blumberg's sign is abdominal rebound tenderness. This is an indication of peritoneal irritation.
A 50-year-old man presents with weakness and abdominal discomfort. Upon questioning for medical history, he acknowledges a lack of sexual desire. He denies any photosensitivity. On physical examination, his liver is enlarged. He has an abnormal skin pigmentation that is located on his face, neck, and elbows; it gives his skin a gray-brown hue. His laboratory results are in the chart. TEST RESULTS REFERENCE RANGE TIBC 275 μg/dL 250 - 375 μg/dL Plasma iron 220 μg/dL 50 - 150 μg/dL Transferrin saturation 90% 20 - 40% Question What is the most likely diagnosis? Answer Choices 1 Hemochromatosis 2 Lead poisoning 3 Wilson's disease 4 Protoporphyria 5 Iron deficiency anemia
Hemochromatosis This patient has signs and symptoms consistent with hemochromatosis. Hemochromatosis is due to an increase in iron within the tissues. The liver is commonly affected, with hemochromatosis; hepatomegaly can be seen. There is an abnormal skin pigmentation that can be seen with hemochromatosis. Plasma iron will be elevated, total iron binding capacity will be normal or low, and transferrin saturation will be elevated. Lead poisoning can cause abdominal pain; however, the laboratory results of this patient are not consistent with lead poisoning. Wilson's disease is due to an accumulation of copper. The liver is also affected with Wilson's disease. The laboratory tests in this scenario do not support a diagnosis of Wilson's disease. Ceruloplasmin is the blood test that should be done in evaluation of a patient for Wilson's disease. Protoporphyria is due to a defect in heme synthesis and metabolism. Photosensitivity is seen with protoporphyria. The lab results are not consistent with iron deficiency anemia. Serum TIBC would likely be increased in iron deficiency anemia, not normal (as in the case here). With iron deficiency anemia there also would be a decrease in serum iron level, not an increase (as is seen in this case).
A 42-year-old man presents with severe abdominal pain in the mid-epigastric region. The pain began 3 days ago with an intensity of 10/10; there is concomitant nausea and vomiting (on 2 occasions). The patient denies any significant past medical history, but he states that he "only" drinks 10 beers a day, and he has been doing so for the past 15 years. BP is 86/58. Pulse is 112, and temp is 101.5F. There is rebound tenderness in the midepigastric area, absent bowel sounds, and a positive Cullen's sign. The rest of the physical exam is within normal limits. In cases of acute pancreatitis, a positive Cullen's sign suggests what complication? Answer Choices 1 Perforated viscus 2 Pancreatic abscess 3 Hemorrhagic pancreatitis 4 Pancreatic ascites 5 Pancreatic pseudocyst
Hemorrhagic pancreatitis The correct response is hemorrhagic pancreatitis. More than 70% of cases of acute pancreatitis are due to alcohol abuse or gallstones. Complications, such as adult respiratory distress syndrome, hemorrhagic pancreatitis, pancreatic abscess, pancreatic pseudocyst, and pancreatic ascites, among others, account for the 10% mortality rate associated with acute pancreatitis. Hemorrhagic pancreatitis can lead to retroperitoneal hemorrhage and widespread tissue necrosis, and it may require surgical intervention or peritoneal lavage. This retroperitoneal bleeding can be responsible for the discoloration or hematoma in the periumbilical area (Cullen's sign).
A 37-year-old woman presents with a persistent flu-like illness, malaise, anorexia, nausea, and vomiting. The patient is a non-smoker; she drinks alcohol moderately, and she has no remarkable travel history. She admits to using parenteral drugs during her college life. Examination shows icteric sclerae, splenomegaly, right upper quadrant tenderness, and dark urine. What has the highest risk of chronic infection in adults? Answer Choices 1 Hepatitis A virus 2 Hepatitis B virus 3 Hepatitis C virus 4 Hepatitis D virus 5 Hepatitis E virus
Hep C Hepatitis C is a viral infection caused by an enveloped RNA virus belonging to the flaviviridae family. It is transmitted parenterally by sharing needles during parenteral drug use as well as from mother to child. The medical staff is also at risk of infection and sexual transmission can also occur. Other groups at risk of infection are those undergoing hemodialysis and recipients of organ transplant before 1992. Blood, blood products, and organ donors have been screened for hepatitis C since 1992 and the risk has been minimized in recipients now. Acute hepatitis C is asymptomatic in 60 - 70% of patients. Chronic infection develops in 55 - 85% of patients, chronic liver disease in 70% of patients, and cirrhosis in 5 - 20% of patients. Combination therapy with pegylated interferon and ribavirin is the treatment of choice resulting in sustained response rates of 40 - 80%. Hepatitis C is the leading indication for liver transplant in the US. Hepatitis A is transmitted by the feco-oral route and it has no chronic form. Once infected, it provides immunity for life. A vaccine is also available for travelers to endemic areas and other susceptible groups. Hepatitis B is also transmitted parenterally and sexually. The risk of chronic infection varies with age; for those over 5 years of age, it is about 2 - 6%. Infants have a risk of chronic infection of 90%, and between 1 - 5 years it is about 30%. Hepatitis D occurs as a co-infection or superinfection in patients infected with hepatitis B. It cannot infect a person by itself because it requires the envelope protein of hepatitis B. Chronic HBV carriers who acquire HDV superinfection usually develop chronic HDV infection. Hepatitis E is transmitted feco-orally, and the most common source is contaminated water. The acute hepatitis is most severe in pregnancy, especially the 3rd trimester. It has no chronic form. Hepatitis A and E are rare in the United States.
A 68-year-old man presents for a routine physical exam. A soft venous hum over the epigastric and umbilical area is noted on auscultation. The venous hum is due to increased collateral circulation resulting from what type of condition? Answer Choices 1 Hepatic cirrhosis 2 Chlamydial perihepatitis 3 Gonococcal perihepatitis 4 Liver tumor 5 Recent liver biopsy
Hepatic cirrhosis A venous hum is due to an increased collateral circulation between the systemic and portal venous system resulting from hepatic cirrhosis. Chlamydial and gonococcal perihepatitis, liver tumor, and a recent liver biopsy cause grating sounds called "friction rubs" along the hepatic and splenic borders of the abdomen.
A 28-year-old woman presents with a 5-day history of progressive jaundice, vomiting, nausea, and malaise. She has developed a fever and has a 2-day history of dark urine. She has no history of intravenous drug use and has had no recent transfusions; she denies any sexual contact in the last 3 months. The patient has recently returned from a trip to Mexico where she consumed various foods from street vendors. Physical examination reveals a jaundiced patient. An abdominal examination is significant for a palpable liver edge 2 cm below the right costal margin and a total liver span of 12 cm below the mid-clavicular line. Blood is drawn for cultures, a complete blood cell count, and liver function tests. Blood studies are significant for a direct bilirubin of 13.0 mg/dL, aspartate aminotransferase of 1,800 U/liter, and alanine aminotransferase of 2,500 U/liter. Blood cultures are negative and serological studies are conclusively diagnostic. What is the most probable cause of the patient's symptoms? Answer Choices 1 Hepatitis A 2 Norwalk virus 3 Rotavirus 4 Adenovirus 5 Parvovirus
Hepatitis A Hepatitis A is a member of the Picornaviridae family; it is a nonenveloped virus approximately 27 - 28 nm in diameter. It is spherical, with a surface structure that suggests icosahedral symmetry. It is a single-stranded RNA virus. Signs and symptoms of an infection include diarrhea, dark urine, jaundice, and flu-like symptoms. Incubation is 15 - 50 days, with an illness duration of 2 weeks - 3 months. The virus is associated with shellfish harvested from contaminated waters, raw produce, uncooked foods, and cooked foods that are not reheated after contact with an infected food handler. Confirmation is based on the presence of anti-hepatitis A antibodies, positive IgM, and increases in serum ALT and bilirubin. Treatment is supportive, and prevention is by vaccination. Norwalk virus is a single-stranded RNA virus that is approximately 29 - 35 nm in diameter with spiky ill-defined outlines. The virus produces gastroenteritis-like symptoms that include nausea, vomiting, and watery/large volume diarrhea. Fever is usually absent. The incubation period is 24 - 48 hours, and the illness duration is 24 - 60 hours. Sources of infection include poorly cooked shellfish and ready-to-eat foods (e.g., salads and sandwiches) touched by infected food handlers. Diagnosis is based on negative stool cultures, stool that is negative for white blood cells, and serological studies for the presence of fourfold or greater increases in antibody titers of Norwalk virus antibodies. Treatment is mainly supportive; bismuth sulfate may be given. Rotavirus is a double-stranded RNA virus belonging to the Reoviridae family. The virus is 70 nm in diameter and appears as a wheel. The virus infection has signs and symptoms that include vomiting, watery diarrhea, and low-grade fever. There may be temporary lactose intolerance. The virus is especially common in infants and children, the elderly, and the immunocompromised. The incubation period is 1 - 3 days, and the duration of illness is 4 - 8 days. The virus is associated with ready-to-eat foods touched by infected food handlers (e.g., salads and fruits) and fecally-contaminated foods. Confirmation is based on the detection of the virus antigen in stool. Stool routine cultures are negative, and stools are negative for the presence of white blood cells. Treatment is supportive in nature. Fluid electrolyte replacement may be needed in severe cases of diarrhea, especially in infants. Adenovirus is a virus with DNA as the genetic material; it is approximately 70 nm in diameter. The virus causes gastroenteritis; symptoms include vomiting, nausea, malaise, diarrhea, headache, and fever. Incubation is 10 - 70 hours, with a duration of 2 - 9 days for symptoms. The organism is associated with ingesting food contaminated with feces and ready-to-eat foods touched by infected food workers. The virus is also found to be transmitted through the ingestion of some shellfish. The virus can be detected antigenically in acute stool samples. Stool staining for white blood cells is negative. Supportive care is the treatment of choice. The disease is usually mild and self-limiting. Parvovirus is a single-stranded DNA virus. Infections with this virus are mostly asymptomatic, although the virus produces erythema infectiosum (fifth disease) in small children (a benign rash appears on the face, trunk, and extremities). A fever also develops. The disease can cause intrauterine infection and fetal death due to the vertical transmittance of the virus from mother to fetus. In individuals with hemoglobinopathies, there can be transient aplastic crisis. Immunodeficient patients can develop chronic anemia.
A 50-year-old man presents with a 3-month history of weakness, fatigue, and abdominal discomfort. Upon further questioning, he acknowledges a lack of sexual desire. He denies any photosensitivity. On physical examination, his liver is enlarged, and his spleen is palpable. He has abnormal skin pigmentation on his face, neck and his elbows and which gives his skin a metallic gray hue. His laboratory results are in the chart. TEST RESULTS REFERENCE RANGE TIBC 275 μg/dL 250 - 350 μg/dL Plasma iron 220 μg/dL 80 - 160 μ/dL Transferring saturation 90% 16 - 57% Question What serious complication is associated with the patient's condition? Answer Choices 1 Hepatocellular carcinoma 2 Bronchogenic carcinoma 3 Pancreatic carcinoma 4 Lymphoma 5 Leukemia
Hepatocellular carcinoma This patient has signs and symptoms consistent with hemochromatosis, which is associated with hepatocellular carcinoma. Hemochromatosis is due to an increase in iron within the tissues. Hemochromatosis is a disorder of iron overload; it could be due to genetic or non-genetic causes. In hereditary hemochromatosis, there is absorption of a few milligrams of iron each day, in excess of need. As such, clinical manifestations often occur only after the age of 40 years, when body iron stores have reached 15 to 40 g. The symptoms of hemochromatosis are nonspecific and include arthralgia; abdominal pain; fatigue, weakness; impotence; weight loss; amenorrhea and early menopause; abnormal skin pigmentation; damage to the pancreas leading to diabetes; cardiomyopathy; and cirrhosis. The liver is commonly affected with hemochromatosis, and hepatosplenomegaly is commonly seen. There is an abnormal skin pigmentation that is seen with hemochromatosis. In hemochromatosis, the plasma iron will be elevated; total iron binding capacity will be normal or low; and transferrin saturation will be elevated. Hepatocellular carcinoma is the most serious complication, and it is a major cause of death in patients with hemochromatosis. A large percentage of patients with hemochromatosis will develop problems with their pancreas; however, the pancreatic pathology commonly seen with hemochromatosis is insulin dependent diabetes, not pancreatic cancer. Bronchogenic carcinoma, pancreatic carcinoma, lymphoma, and leukemia are not known complications of hemochromatosis.
A 55-year-old African-American man presents as febrile with massive swelling of the abdomen and diarrhea. He has a 20-year history of heavy alcohol use. A fluid wave is elicited on physical examination of the abdomen by striking one flank and feeling the transmitted wave on the opposite flank. Question In what case would ascitic fluid analysis suggest cirrhosis as a cause of ascites in this patient? Answer Choices 1 If the acidic fluid and leukocyte count is more than 500/mm3 2 If the fluid has a protein concentration below 3 g/dL 3 If there is a large number of red blood cells in the fluid 4 If fluid is designated as a exudate 5 If the Serum Ascites Albumin Gradient is lower than 1.1 g/dL
If the fluid has a protein concentration below 3 g/dL The correct response is if the fluid protein concentration below 3/dL. After the diagnosis of ascites is made by physical examination, all patients with new-onset ascites should undergo abdominal paracentesis and ascitic fluid analysis. The most important tests to order for fluid analysis include protein concentrationand cell count. Fluids with protein concentration above 3 g/dL are designated as exudates. Those with values below 3 g/dL are designated as transudates. Diseases usually associated with transudates include congestive heart failure, cirrhosis, constrictive pericarditis, inferior vena cava obstruction, hypoalbuminemia, Meigs syndrome, and some cases of nephrotic syndrome. The amount of albumin in the ascitic fluid compared to the serum albumin (the Serum Ascites Albumin Gradient, SAAG) can be indicative of the cause of ascites. Ascites related to hypertension, cirrhosis, or congestive heart failure generally shows a SAAG greater than 1.1 g/dL. Exudates are more commonly seen with peritoneal neoplasm, pancreatic ascites, myxedema, and tuberculous peritonitis. A large number of red blood cells in the fluid or grossly bloody ascites suggests a diagnosis of neoplasm. An acidic fluid and leukocyte count of more than 500/mm3 strongly suggests a peritoneal infection or inflammatory process. Other tests that should be ordered in the appropriate clinical setting include cytologic examination, lactic dehydrogenase (LDH), specific tumor markers, glucose, and cultures for bacteria, mycobacteria, and fungi.
A 51-year-old woman presents with a long history of blurry vision, palpitations, and weakness. Her lab results are as follows: TEST RESULTS REFERENCE RANGE Calcium 8.9 mg/dL 8.4-10.2 mg/dL Potassium 3.8 mEq/L 3.5-5.0 mEq/L Sodium 144 mEq/L 135-145 mEq/L Glucose 41 mg/dL 70-110 mg/dL You suspect a pancreatic islet cell tumor. What is the most likely diagnosis? Answer Choices 1 VIPoma 2 Insulinoma 3 Gastrinoma 4 Glucagonoma 5 Somatostatinoma
Insulinoma Her low blood glucose levels indicate an insulinoma. The endocrine portion of the pancreas is housed in the islets of Langerhans. Tumors that arise from this area are referred to as islet cell tumors. The specific type of tumor depends on the cell type involved. Typically, the tumor is named based on the hormone it is secreting. An insulinoma is an islet cell tumor. Insulin is produced by the beta cells of the islets of Langerhans. Therefore, an insulinoma is a tumor of the beta cells. An insulinoma is the most frequently occurring islet cell tumor. Hypoglycemia, secondary to the insulin secretion, is a symptom. Verner-Morrison syndrome is due to a VIPoma. This is an endocrine tumor of the pancreatic islets of Langerhans. It secretes vasoactive intestinal polypeptide. Vasoactive intestinal polypeptide is produced by the D1 cells of the islets of Langerhans. Therefore, a VIPoma is a tumor of the D1 cells. Other names for a VIPoma include pancreatic cholera and WDHA-syndrome (watery diarrhea, hypokalemia and achlorhydria). Gastrinomas are most often due to islet cell tumors. On occasion, gastrinomas can arise outside of the pancreas. Peptic ulcers, secondary to the excessive gastrin secretion, are frequent. Gastrinomas and peptic ulcerations are referred to as the Zollinger-Ellison syndrome. A glucagonoma is an islet cell tumor. Glucagon is produced by the alpha cells of the islets of Langerhans. Therefore, a glucagonoma is a tumor of the alpha cells. Diabetes can be seen with a glucagonoma. Other symptoms include a skin rash, weight loss, and anemia. A glucagonoma is a rare islet cell tumor. A somatostatinoma is an islet cell tumor. Somatostatin is produced by the delta cells of the islets of Langerhans. Therefore, a somatostatinoma is a tumor of the delta cells. Somatostatinoma is a rare islet cell tumor.
A 54-year-old woman presents to the emergency department with nausea, vomiting, right-upper quadrant abdominal pain, fever, and jaundice starting 4 hours ago. During the last 6 months, she suffered several bouts of upper-abdominal pain accompanied by nausea, vomiting, and occasional jaundice, for which medical attention was sought. Her past medical history includes hyperlipidemia, for which she first took simvastatin; she switched to cholestyramine because of side effects. Vital signs on admission are as follows: BP 110 / 80 mm Hg, HR 90 bpm, RR 20 rpm, temperature 38.1°C (100.6 F). She is alert and oriented, and mildly jaundiced. Her right-upper abdomen is diffusely tender to palpation. An upper abdominal ultrasound is performed and reveals a thickening and calcifications of the gallbladder wall, but there are no signs of air within the peritoneal cavity or the bile ducts. What is the most appropriate next step in management? Answer Choices 1 Intravenous hydration and antibiotics 2 Endoscopic papillotomy 3 Give analgesics and refer for elective cholecystectomy 4 Magnetic resonance cholangiography to look for choledocholithiasis 5 Emergency cholecystectomy
Intravenous hydration and antibiotics The presence of fever, jaundice, and right-upper quadrant pain defines Charcot's triad, which is the classical presentation of acute cholangitis. Reynolds' pentad consists of Charcot's triad plus sepsis/shock and mental status changes. 95% of patients presenting with these syndromes have common duct stones. In most cases, there is a favorable response to conservative treatment, which consists of interrupting oral feeding, analgesia, intravenous hydration, and antibiotics. The following antibiotic schemes can be used: A third-generation cephalosporin + an aminoglycoside Piperacillin or ampicillin + metronidazole + an aminoglycoside Monotherapy with imipenem, meropenem, mezlocillin, ampicillin-sulbactam, ticarcillin-clavulanate, or piperacillin-tazobactam After clinical improvement, cholecystectomy is undertaken in the first few days after the initial hospitalization in order to avoid recurrence. Surgery is warranted because the recurrence rate is as high as 10% per year in patients whose gallbladders are not removed. The use of cholestyramine is another risk factor for recurrence in this patient, and gallbladder calcifications (porcelain gallbladder) are thought to pose a risk of malignization and are considered an indication for cholecystectomy. When the disease progresses despite the initial conservative treatment (i.e., there is worsening fever, leukocytosis, abdominal pain, and guarding), emergency biliary drainage is warranted because of the possibility of gallbladder perforation or gangrene. Some possible approaches are cholecystectomy (conventional or laparoscopic), cholecystostomy, or percutaneous drainage. The latter 2 are usually reserved for sicker patients, who are less likely to tolerate surgery well.
A 58-year-old man presents with a 1-day history of severe abdominal pain, nausea, and vomiting. He initially thought he had some indigestion, with pain located in the epigastric region, and tried some calcium carbonate (Tums) with no relief. The pain and vomiting progressed through the night and kept him from sleeping and going to work. He feels the pain boring through to his back. He denies hematemesis, fever, diarrhea, out-of-the-country travel, and contact with sick people. Prior to onset of pain, he reports good health. He has no known medical conditions and takes no medications. He has had no surgeries. He smokes cigarettes (1ppd x 40 years), admits "moderate" alcohol use, and denies drug use. He is married and works as a welder. Vitals are: BP: 102/56 mmHg; HR: 116bpm; RR: 15; Temp: 98.9F; O2Sat: 95% on room air. On physical exam, the patient appears uncomfortable on the exam table and grimaces when changing position for exam. He is cooperative, alert, and oriented. Abnormal physical exam findings include: abdomen distended, decreased bowel sounds, and tender epigastric region, with guarding. He is tachycardic. No jaundice is noted. The remainder of the exam is normal. This patient's test results are shown in the table. Alk Phosphatase150 (50-136) Amylase 272 (20-110) Lipase 290 (0-160) Question What pharmacologic treatment is the most important intervention for this patient's likely condition? Answer Choices 1 Ertapenem 2 Hyoscyamine 3 Lactated Ringer's 4 Pancrelipase 5 Promethazine
LR This patient is presenting with an episode of acute pancreatitis. Pancreatitis is characterized by epigastric pain, nausea, and vomiting. Many other acute abdomen conditions may present similarly. Significant elevations of lipase and amylase, as well as CT evidence of pancreatic inflammation, fluid collections, and/or necrosis, will establish the diagnosis. Pancreatitis can be acute or chronic, and those with chronic disease may have periodic acute flares. In acute pancreatitis, early fluid resuscitation is one of the "few medical interventions that appears to affect outcome," so (of the choices listed) administering lactated Ringer's (or saline) would be the best answer for this patient. Also, ensuring the patient has "nothing by mouth" (NPO) until symptoms decrease is standard treatment. Ertapenem is a carbapenem antibiotic, indicated for complicated intra-abdominal, skin, and urinary tract infections. Although this patient's white blood cell count (WBC) is elevated, the primary process is not infectious. The WBC increases in inflammatory states. Rarely are antibiotics needed in acute pancreatitis. Hyoscyamine is an anticholinergic medication, with a wide range of uses, including many gastrointestinal conditions. It may be used for reducing spasm in irritable bowel syndrome and for biliary colic. Before the test results pointed to pancreatitis, as in this patient's diagnosis, biliary colic and obstruction would be on the differential. However, hyoscyamine would not be a primary treatment for pancreatitis. Pancrelipase is a digestive enzyme, taken with meals, and indicated for pancreatic insufficiency, a common complication of many years of chronic pancreatitis. This patient may be at risk for future pancreatitic disease, but the pancrelipase would not benefit him at this time. Promethazine is a common antiemetic with antihistamine and anticholinergic properties. Although it may be reasonable to consider antiemetic treatment for a patient with acute pancreatitis, it is an adjunct treatment and does not have a vital impact on the course of the disorder.
A patient with a history of alcohol abuse presents with a histopathological report; he is asking about the 'bodies found in his liver cells'. What type of bodies is he talking about? Answer Choices 1 Russell bodies (protein inclusions) 2 Lewy bodies (protein aggregates) 3 Negri bodies (eosinophilic inclusions) 4 Heinz bodies (hemoglobin precipitations) 5 Mallory bodies (hyaline inclusions) 6 Guarnieri bodies (intracytoplasmic inclusions)
Mallory bodies (alcoholic hyaline) are cytoplasmic inclusions found most often in alcoholic hepatitis. They are abnormal aggregations of cytoskeletal proteins. Russell bodies are found in reactive or malignant plasma cells, not in liver cells. They represent excess immunoglobulin. Lewy bodies are abnormal aggregates of protein inside nerve cells in Parkinson's disease (PD) and some other degenerative brain disorders. They are not found in liver cells. Negri bodies are eosinophyllic, viral, intracytoplasmic inclusions found in hippocampal nerve cells. They are pathognomonic for rabies infection. Heinz bodies are inclusions within red blood cells that are composed of denatured hemoglobin. Although they can be found in liver disease, as well as alpha thalassemia and glucose-6-phosphate dehydrogenase, they are not found in liver cells. Guarnieri bodies, or B-type inclusions, are the sites of viral replication; they are found in all poxvirus-infected epithelial cells, but not in the hepatocytes.
A 33-year-old woman presents seeking advice. She is concerned regarding the appearance of spider angiomas that are present on her trunk and face. She states that spider angiomas appeared during pregnancy and that they persist now, 9 months after delivery. Before pregnancy, she was treated with imipramine because of depression and she is currently still taking this drug. She denies alcohol intake, however she tells you that 3 years ago, she had acute infection with hepatitis C virus. She was also frequently treated with flucloxacillin during the past couple of years for recurrent respiratory tract infections. In addition, she started taking oral contraceptives after delivery. The remainder of her personal history is unremarkable. Physical examination reveals the presence of multiple spider angiomas on the patient's face, forearms, and back. The remainder of the patient's general physical findings are unremarkable. Routine laboratory analyses reveal normal AST (26 IU/l, reference values 8 to 27 IU/l) and ALT (22 IU/l, reference value 8 to 23 IU/l) levels. Alkaline phosphatase level is normal (43 IU/l, reference value 23-71 IU/l) and other routine laboratory analyses reveal no abnormalities. HbSAg, HbeAg, and antiHbc antibodies are not present in patient's serum. IgG antibodies to hepatitis C are present, but testing for hepatitis C virus (HCV) does not reveal the presence of HCV RNA in patient's serum. Question What is the most likely cause of the patient's spider angiomas? Answer Choices 1 Flucloxacillin-induced hepatitis 2 Imipramine-induced hepatitis 3 Oral contraceptives 4 Chronic hepatitis caused by hepatitis C virus 5 Pregnancy-related appearance
OCP The most likely cause of this patient's spider angiomas are Oral contraceptives. In conditions when estrogen hormones are present in excess such as during pregnancy, contraceptive intake, or liver disease (due to decreased degradation of estrogens), the surplus of estrogens cause the appearance of spider nevi. In pregnant women, spider angiomas appear from the second to fifth month of pregnancy and disappear in a short period of time (within 2 months) after the delivery. Since in the presented case, spider angiomas are present for a longer period of time, it is unlikely that they are pregnancy-related. In untreated patients with acute Hepatitis C, disease evolves towards chronicity in about 80% of patients, while only about 20% of patients recover completely. Patients who have chronic hepatitis C also have HCV RNA in their serum, but those who have recovered completely have no HCV RNA in serum and have IgG antibodies against hepatitis C virus as the marker of past hepatitis C infection. As the presented patient satisfies criteria for the patient who had hepatitis C in the past, her spider angiomas are not related to chronic hepatitis C infection. Imipramine seldom causes liver damage. In addition, hepatitis caused by imipramine administration is of a cholestatic type, i.e. it is associated with increased serum levels of transaminases and alkaline phosphatase. Since the alkaline phosphatase level in the presented patient is normal, imipramine-induced hepatitis is not likely. Similar to imipramine, hepatitis caused by flucloxacillin administration is also of a cholestatic type. For the reasons presented above, the possibility of flucloxacillin-induced hepatitis is also unlikely.
A 45-year-old woman presents with yellow discoloration of the skin, clay-colored stool, and dark urine. She is obese, but otherwise healthy. She has long-standing digestive troubles and an intolerance to fatty meals for which she takes antispasmodic medications. On examination, there is mild tenderness in the upper right abdominal quadrant; serum bilirubin shows elevated total and conjugated types, as well as normal unconjugated bilirubin. Question What is the most likely cause of her jaundice? Answer Choices 1 Obstructive reasons 2 Hemolytic anemia 3 Hepatic enzyme deficiency 4 Hepatitis 5 Drug-induced hemolysis
Obstructive reasons With obstructive jaundice, bilirubin is formed and conjugated normally but the process of biliary drainage is impaired; an increase in serum conjugated bilirubin is seen. This can be due to a stone or tumor in the bile ducts, liver, or pancreas. Jaundice (or icterus) is a yellowish discoloration of the skin and mucous membranes due to increased serum bilirubin. Jaundice can occur due to a deficit in the bilirubin clearance pathway. Normally, bilirubin is unconjugated in the blood; it undergoes conjugation in the liver and is secreted in the bile. Increased unconjugated serum bilirubin occurs due to increased hemolysis, and it is usually mild. When there is a defect in liver conjugation due to congenital or acquired reasons (e.g., hepatitis), conjugation decreases; this results in increased unconjugated serum bilirubin. In hepatitis, there is also some defective biliary drainage, so both unconjugated and conjugated bilirubin increases. There is also clay-colored stool, dark urine, and itching.
A 49-year-old Caucasian woman presents with a 2-month history of worsening abdominal pain. The patient describes the pain as crampy, unrelenting abdominal pain that begins in the right upper and middle quadrant of the abdomen; very occasionally, it will go into right shoulder area. The pain usually occurs shortly after especially heavy meals, usually within 30 minutes; it then eventually subsides, although this takes anywhere from 60 - 120 minutes. Physical examination reveals significant right upper quadrant tenderness and rigidity with a palpable, firm mass. Ultrasound performed in the office reveals complete calcification of the gallbladder wall; there is a hyperechoic semilunar appearance with posterior acoustic shadowing. Question A symptomatic patient as described above with such severe calcification as seen on ultrasound would most likely be treated via what method? Answer Choices 1 Reassurance only 2 Periodic ultrasound imaging study 3 Open cholecystectomy 4 Laparoscopic cholecystectomy 5 Percutaneous biliary stent placement
Open cholecystectomy Patients withsymptomatic porcelain gallbladdersare much more commonly female than male, with the typical age range being from 38 - 70 years old. In general, patients will describe a history of biliary type pain. Diagnostically, an ultrasound or CT scan will most efficiently give the visualization of the calcification necessary to confirm a porcelain gallbladder.Those found to have complete mural calcification (complete type) generally will need to have it treated with an open cholecystectomy. This is because the mucosal calcification that creates the porcelain characteristic makes the gallbladder very thick and fibrotic, which in turn makes it potentially difficult to grab by forceps and dissect out in a laparoscopic approach. There have been recent studies suggesting a laparoscopic cholecystectomy would be acceptable in patients who have evidence of a long cystic duct and biliary anatomy that is well-defined perioperatively, as well as in those who have evidence of a less advanced form of porcelain gallbladder, although there is no confirmed change of recommended treatment plan at this time. Anytime a porcelain gallbladder is found, there has to be a heightened alert for the suspicion of gallbladder carcinoma. The more advanced and severe the calcification is of the gallbladder, the heightened potential of it being malignant. Reassurance only and periodic ultrasounds would not be appropriate in this scenario. This approach may be followed in some part in a patient with less severe signs and symptoms. Percutaneous biliary stent placement is not the appropriate choice for this clinical scenario, as it is not treating the main pathology being presented.
A 64-year-old man presents with a 3-week history of weight loss, abdominal pain, loss of appetite, nausea, weakness, fatigue, vomiting, diarrhea, indigestion, back pain, and depression. Past medical history is negative. Social history includes smoking 2 packs of cigarettes a day for 35 years. Clinical exam includes abdominal pain in the upper quadrants and pallor. Fecal fat test is positive. What is the most likely diagnosis? Answer Choices 1 Acute pancreatitis 2 Chronic pancreatitis 3 Pancreatic carcinoma 4 Pacreatic abscess 5 Insulinoma
Pancreatic carcinoma is the correct response. Itis the 4th most common cancer causing death in the U.S. The disease is more common in men, especially those between 60 and 70 years. The cause is unknown; however, the incidence is greater in smokers. High fat diet and chemical exposures may increase the risk. Symptoms include weight loss, abdominal pain, loss of appetite, jaundice, nausea, weakness, fatigue, vomiting, diarrhea, indigestion, back pain, stools (clay colored), pallor, and depression. Tests should include a pancreatic biopsy, an abdominal CT scan, and abdominal ultrasound. Only 20% of the tumors are operable at the time of diagnosis. Palliation is generally the treatment, along with chemotherapy and radiation. The chief causes of acute pancreatitis in adults are gallstones, other biliary diseases, or alcohol use. Viral infection (mumps, Coxsackie B, mycoplasma pneumonia, and Campylobacter), injury, pancreatic or common bile duct surgical procedures, and certain medications (especially estrogens, corticosteroids, thiazide diuretics, acetaminophen, tetracycline), are other causes. After the triggering event, the process continues with autodigestion that causes swelling, hemorrhage, and damage to the blood vessels. An attack may last for 48 hours. Symptoms include abdominal pain (mainly located in the upper abdomen) nausea, vomiting, weakness, sweating, anxiety, fever, clammy skin, and mild jaundice. General examination may show a low blood pressure and a heart rate above 90. Most cases resolve within 1 week with supportive measures such as fluid replacement. Chronic pancreatitis is caused by alcohol abuse, hemochromatosis (a condition of excess iron in the blood), and other unknown factors. Inflammation and fibrosis cause the destruction of functioning glandular tissue in the pancreas. This results in an inability to properly digest fat due to a lack of pancreatic enzymes. The production of insulin is also affected. Symptoms include abdominal pain (mainly in the upper abdomen), nausea, vomiting, weight loss, and fatty stools. Additional symptoms may include swelling (overall), clay-colored stools, and abdominal indigestion. Pancreatic abscess occurs in 5 to 10% of people with acute pancreatitis. An abscess may be caused by inadequate drainage of a pancreatic pseudocyst; a complication associated with pancreatitis. Symptoms include fever, chills, abdominal pain, and abdominal mass. Physical exam will show signs of pancreatitis, and tests should include abdominal CT and ultrasound. Treatment will include laparotomy with drainage and possible resection of dead tissue. Insulinomas are generally benign tumors of the insulin-secreting cells of the pancreas, which secrete excess amounts of insulin. Risk factors include a prior history of multiple endocrine neoplasia Type I (MEN I). Symptoms include sweating, tremor, rapid heart rate, anxiety, hunger, dizziness, headache, clouding of vision, confusion, behavioral changes, convulsions, and loss of consciousness. Surgery is the treatment of choice to remove the tumor. If the tumor is not found during surgery, diazoxide may be given. A diuretic is always given with this medication to avoid retaining too much salt.
A 42-year-old morbidly obese woman is referred back to her primary care provider to supervise her weight loss program. A surgeon evaluated her for a 'gallbladder attack' 1 month ago, which then resolved; she currently denies abdominal pain. When examined via ultrasound, she still has visible stones in the gallbladder. She declined to have surgery until she achieves her initial weight loss goal of 50 pounds. She is on a supervised healthy diet of about 1200 kilocalories daily; she exercises 1 hour a day 6 days per week. She is losing about 15 - 16 pounds a month. Question What intervention would be most effective at preventing the recurrence of her gallbladder "attack" until she achieves her weight loss goal? Answer Choices 1 Prescribe orlistat 2 Prescribe ursodiol 3 Recommend over-the-counter omeprazole 4 Reduce patient's exercise 5 Restrict caloric consumption further
Prescribe Ursodiol Both morbid obesity and rapid weight loss are risk factors for development of cholecystitis. Gallstones may be present within the gallbladder and remain asymptomatic, or the gallbladder walls may become inflamed, resulting in cholecystitis. Ursodiol is approved for prevention of gallstones in obesity patients with rapid weight loss. It would be the best choice in health maintenance and delaying or preventing need for surgical cholecystectomy. Orlistat is a prescription medication approved for treatment of obesity. Its mechanism is to block fat absorption. It does not have a direct role in health maintenance or prevention of cholecystitis or cholelithiasis. In fact, it has pronounced gastrointestinal side effects. Over-the-counter omeprazole is a commonly used proton-pump-inhibitor (PPI); it is helpful in acid lowering in the stomach and treatment of gastroesophageal reflux disease. It has not been shown to help gallbladder disease, and there is some evidence that PPIs may worsen it. Reducing the patient's exercise may aggravate her condition and negatively impact her weight loss. Exercise has an inverse relationship with cholecystitis and rates of cholecystectomies, so this patient should be encouraged to continue frequent exercise. This patient should not be instructed to restrict caloric consumption further. She is already losing weight quite rapidly at around 4 pounds per week. Her daily caloric intake is low; recommending further restriction will likely discourage her, and there will not be any benefit in regard to her gallbladder disease.
A 45-year-old man is admitted to the medical floor of a hospital with increasing jaundice, swollen legs, and episodes of disorientation for the last several weeks. His abdomen is distended and he looks ill. According to the family, he is an alcoholic and has recently been diagnosed with cirrhosis of liver. On examination, he has a temperature of 99 F, BP 100/72 mm Hg, and his pulse is 86/minute. Sclerae are icteric, and he has 2+ pitting pedal edema. Lungs are clear, and heart sounds are normal. Abdomen is distended with moderate ascites, caput medusae, and no tenderness. Liver and spleen are not palpable. He is awake but drowsy and oriented to person and place but not to time. He does have a fine tremor in his hands. Labs show WBC 8000/uL, platelets 100,000/uL, Hb 12g%, AST 76 U/L, ALT 56 U/L, AP 62 U/L, and ammonia is 124 mg/dL. Question At this time which of the following is most important to avoid full decompensation into hepatic encephalopathy? Answer Choices 1 Increase dietary protein 2 Prevent constipation 3 Sedate the patient to avoid injury from disorientation 4 Start IV antibiotics empirically anticipating infection 5 Add thiamine and folic acid for nutritional support
Prevent Constipation Explanation This patient is already in the initial phases of hepatic encephalopathy due to alcoholic liver disease as evidenced by the disorientation and tremors, also known as asterixis. The primary cause of hepatic encephalopathy is unclear. Metabolic abnormalities due to liver dysfunction, resulting in a spectrum of neuropsychiatric signs and symptoms, are seen. High levels of ammonia are found in the blood. Constipation causes increased ammonia production and absorption due to prolonged intestinal contact and aggravates the condition. Lactulose should be administered frequently to eliminate the ammonia in the stool. It is an indigestible sugar that acts as an osmotic laxative by increasing the water content of the stool and promoting bowel movements. It is digested by the colonic bacteria, and the acidic remains convert ammonia into ammonium ions in the colon, which are then excreted in the stool. Oral antibiotics can lower blood ammonia levels by decreasing ammonia production and absorption. The commonly used ones are neomycin, metronidazole, vancomycin, and, lately, rifaximin. The last three are better tolerated than neomycin. However antibiotics have their side effects and can cause bacterial overgrowth syndromes. Their main use continues to be in patients who cannot tolerate disaccharides like lactulose. Acarbose and fermentable fiber can also cause decrease in intestinal ammonia production and absorption. Newer studies with sodium benzoate are ongoing. Benzoate and glycine react to form hippurate, and for every mole of benzoate utilized this way, one mole of nitrogen is excreted in the urine, thereby enhancing ammonia metabolism. This, however, still needs to be studied further to be used widely. Studies with ornithine-aspartate are also being done as a stimulator of ammonia metabolism. All the products mentioned are yet to replace lactulose as the first line of treatment but are potentially useful once more studies are done. Other precipitating factors for hepatic encephalopathy include azotemia, hypokalemia, gastrointestinal bleeding, high protein diet, alkalosis, infection, sedatives, and other hepatotoxic agents. High protein diet is a contraindication in this condition, as protein catabolism causes increase in ammonia levels. Daily protein should be restricted to 40 g/day. Sedation of the patient should also be avoided, since sedatives cause cerebral depression and worsening of encephalopathy. These drugs are also not metabolized adequately by the diseased liver. Though infection is an important precipitating factor for hepatic encephalopathy, empiric treatment is not recommended. However, early and adequate treatment of an infection should be done, especially for spontaneous bacterial peritonitis. In fact multiple randomized control trials have been done regarding antibiotic prophylaxis for SBP, and they have shown not only a decrease in bacterial infections but also a significant reduction in mortality. Prophylaxis is recommended in cirrhotic patients with risk factors for SBP like GI bleeding, prior history of SBP, and low ascitic fluid protein. Thiamine and folic acid should be added for nutritional support to all alcoholic patients, since they are malnourished and vitamin depleted. However, this will not change the outcome in hepatic encephalopathy as quickly or as much as avoiding constipation.
A middle-aged woman presents with elevated cholestatic liver enzyme levels. She is not taking any medications, does not drink alcohol, and does not complain of abdominal pain. She has not had any previous biliary tract surgery. Which of the following is the most likely diagnosis? A primary biliary cirrhosis B pancreatitis C cholecystitis D fatty liver
Primary Biliary Cirrhosis A Primary biliary cirrhosis affects women typically between ages 40 and 60. It is often discovered incidentally when the serum alkaline phosphatase level is found to be elevated. Many patients do not have pain, which is more common in cholecystitis or pancreatitis. Primary sclerosing cholangitis is more likely to occur in a patient with known inflammatory bowel disease.
A 32-year-old woman presents with fatigue, pruritus, and yellowing of the white of her eyes. Investigations reveal anti-mitochondrial antibodies and elevated alkaline phosphatase (ALP), γ -glutamyl transpeptidase (GGTP) levels. Primary biliary cirrhosis is diagnosed, and she is treated with ursodeoxycholic acid, corticosteroids, and cholestyramine. 1 month later, she returns with abdominal discomfort and constipation. Her symptoms cease when she stops taking cholestyramine. However, the itching is severe when the drug is stopped. Question In order to relieve this patient's itching, what drug may be used as an alternative to cholestyramine? Answer Choices 1 Isoniazid 2 Ethambutol 3 Pyrazinamide 4 Streptomycin 5 Rifampin
Rifampin Rifampin has been found to relieve pruritus in biliary disease. Rifampin is an antimycobacterial agent that can be used in patients who cannot tolerate cholestyramine. It is started at a dose of 150 mg per day and may be gradually increased to 600 mg per day. The mechanism of action of rifampin in pruritus of biliary disease is explained as follows: Being a strong enzyme inducer of the microsomal enzyme inducer system, it promotes the metabolism of endogenous pruritogenic compounds. It competes with the uptake of bile salts in the hepatocytes. It may alter the synthesis of secondary bile acids in the intestinal lumen by its antimicrobial action and thus reduce the amount of lithocholic acid. The other choices are all antimycobacterial agents with no role in pruritus of biliary disease.
A 45-year-old woman presents for initial history and physical at her new primary care provider's office. She is currently asymptomatic, but based on her sexual history with multiple partners, you decide to obtain screening hepatitis lab tests in addition to other screening tests. The following results are obtained: Hep A Ab- negative HBcAb-positive HBeAg-negative HBsAg-positive HBsAb-positive Hep C Ab-positive Hep D Ab-negative Question What is the most likely diagnosis based on these results? Answer Choices 1 She has not been infected with or been vaccinated against Hepatitis B virus but has been exposed to Hepatitis C 2 She has been vaccinated against Hepatitis B in the past but is not currently infected with Hepatitis B nor exposed to Hepatitis C 3 She has had Hepatitis B infection in the past, is currently infected with Hepatitis B, and has been exposed to Hepatitis C 4 She has had Hepatitis B infection in the past but is not currently infected and has been exposed to Hepatitis C 5 She has been vaccinated against Hepatitis B but has not been infected with it and has been exposed to Hepatitis C
She has had Hepatitis B infection in the past, is currently infected with Hepatitis B, and has been exposed to Hepatitis C Explanation: HBcAb, HBsAg, and HBsAb become positive with Hepatitis B virus infection, but over time HBsAg will clear from the serum only if the patient is not chronically infected. These results demonstrate chronic infection, as all three are positive. She also shows exposure to Hepatitis C, as the surface antibody is positive. The remaining answers are incorrect because vaccination alone will cause only the HBsAb to become positive. All Hepatitis B results would be negative if the patient had not had Hepatitis B virus infection or been vaccinated for Hepatitis B virus. Also, only HBsAb would be positive if the patient was never infected but had received the vaccine.
What is the definitive treatment for acute cholecystitis? Answer Choices 1 Antibiotics 2 Antibiotics and observation 3 Surgery 4 Cholecystostomy 5 Observation alone
Surgery The definitive treatment for acute cholecystitis is surgery. Antibiotics and cholecystostomy may be temporizing maneuvers to calm down an acute situation or to temporize a patient that is otherwise too ill to undergo surgery, but the only treatment that will cure the pathology causing the problem is surgery, be it an open cholecystectomy or a laparoscopic cholecystectomy.
The patient is a 47-year-old male who presents to an urgicare center with 18 hours of abdominal pain, nausea, vomiting, and chills. He is a single construction worker, denies smoking, and has at least a 10-year history of drinking 2-4 alcoholic beverages daily. A series of lab work is performed on the patient to evaluate his abdominal pain prior to abdominal imaging. Question What laboratory results would be most indicative of the patient suffering from acute pancreatitis? Answer Choices 1 serum amylase 310 U/L and serum lipase 760 U/L 2 serum amylase 250 U/L and serum lipase 110 U/L 3 serum aspartate aminotransferase 32 U/L and serum alanine aminotransferase 29 U/L 4 serum white blood cell count 14,000/mm3 and serum total bilirubin 1.8 mg/dl 5 serum aspartate aminotransferase 120 U/L and serum alanine aminotransferase 40 U/L
The correct answer is a serum amylase of 310 U/L and a serum lipase of 760 U/L, as these levels are significantly elevated, which is indicative of acute pancreatitis. The normal range for serum amylase is 30-220 U/L, and the normal range for serum lipase is 0-160 U/L. Both levels being significantly elevated is typically seen in a patient with acute pancreatitis. Serum lipase is both more sensitive and more specific than serum amylase for diagnosis of acute pancreatitis, but more so when the serum lipase is at least 3 times the normal level (as is with this case). However, various other biliary and intestinal diseases can also alter these lab results. Blood work results with a serum amylase of 250 U/L and a serum lipase of 110 U/L do not indicate acute pancreatitis. The serum amylase is slightly elevated, but the serum lipase is normal. This can occur for many gastrointestinal-related disorders and renal failure, but these lab results could also commonly be found in a patient with chronic (rather than acute) pancreatitis. Lab results of serum aspartate aminotransferase (AST) 32 U/L and serum alanine aminotransferase (ALT) 29 U/L are actually normal results for these particular lab tests. The AST and ALT can be used to identify hepatocellular diseases of the liver. However, mildly increased levels can also be seen in patients with acute pancreatitis. Lab results of serum aspartate aminotransferase 120 U/L and serum alanine aminotransferase 40 U/L can be seen in patients with various hepatocellular levels as both are increased from normal. These results are possible in a patient with acute pancreatitis, but are not the most indicative of the answer choices. Cirrhosis is a disease of the liver that can often occur in patients with large alcohol intake and often causes lab results such as these. Specifically, results that show an AST level that is 3 times that of the ALT level. Lab results of serum white blood cell count 14,000/mm3 and serum total bilirubin 1.8 mg/dl could be seen with various infections separately. However, the combination of both results limits the possible disorders. The white blood cell count could be elevated in a patient with acute pancreatitis, and the serum bilirubin could also be elevated in patient with acute pancreatitis when it is associated with alcoholic hepatitis. However, these results are not the MOST indicative of acute pancreatitis when compared to the other options.
A 40-year-old man presents with right upper quadrant pain, generalized weakness, and weight loss. He gives a past history of hepatitis B infection. On examination, he has hepatomegaly; there are arterial bruits, ascites, jaundice, and signs of cirrhosis. What investigation would you value most in the diagnosis of this patient? Answer Choices 1 Human chorionic gonadotrophin 2 α-fetoprotein 3 Carcinoembryonic antigen 4 Alkaline phosphate 5 S-100 antigen
The history and the findings are congruent with hepatocellular carcinoma. α-fetoprotein is the major tumor marker for hepatocellular carcinoma, and is elevated in over 70% of patients with this disease. High levels of this enzyme carry a poor prognosis. Human chorionic gonadotropin is raised in cases of hydatidiform moles and chorionic carcinoma. Carcinoembryonic antigen (CEA) is useful in following the progression of gastrointestinal neoplasms, but it is not of diagnostic value. A rise in alkaline phosphatase indicates cholestatic liver disease, bone diseases (e.g., Paget's), osteomalacia, hyperparathyroidism, or renal failure. It may also be elevated in growing children.
A 60-year-old man presents with a 2-week history of nausea and several episodes of epigastric pain. The pain is characteristically dull and lasts for several hours. His symptoms are not worsened by any particular food or activity. What is the most appropriate initial test? Answer Choices 1 Upper endoscopy 2 Gastrointestinal series 3 CT Scan of the abdomen 4 Ultrasonography of the upper abdomen 5 MRI of the abdomen
Ultrasonography of the upper abdomen Ultrasonography (USG) of the upper abdomen is the 1st choice in the investigation of acute or subacute abdominal pain. A common cause of acute abdominal pain in the elderly is cholecystitis, which can be investigated through USG. Changes occurring in the biliary system because of aging make older patients susceptible to cholecystitis, the most common indication for surgery in this population. Differential diagnosis would include diverticulitis (the most common cause of severe abdominal pain in patients over the age of 50), as well as bowel obstruction, pancreatitis, and peptic ulcer disease. Catastrophic conditions (e.g., abdominal aortic aneurysm rupture and mesenteric ischemia) must also be considered. This patient's symptoms are not very intense (his symptoms have been present for 2 weeks) and, therefore, do not necessitate urgent invasive investigations. If ultrasonography of the upper abdomen shows no significant findings, then a number of other tests can be performed (e.g., an upper endoscopy to rule out peptic ulcer disease, a gastrointestinal series to rule out bowel obstruction, and a CT scan or MRI of the abdomen to rule out pancreatitis or diverticulitis); any other undetermined pathology may be undertaken to exclude other possibilities.
A 52-year-old man presents with vomiting and epigastric distress for the past few hours. He has been drinking alcohol for over 20 years, and he has been a moderate-to-heavy drinker. 5 years ago, he was diagnosed with a "gastric/duodenal ulcer", for which he has been taking cimetidine and antacids. The pain now radiates towards the left along the costal margin. He has noticed his appetite has been reduced lately, and his stools are bulky and foul smelling. His friends have commented on his sickly look and weight loss. What is the most likely diagnosis? Answer Choices 1 Chronic pancreatitis 2 Acute recurrent pancreatitis 3 Myocardial infarction 4 Acute cholecystitis 5 Left sided ureteric colic
chronic pancreatitis Chronic pancreatitis is common in alcoholics when pancreatic proteins become denatured and cause destruction of glandular and ductal tissue in the pancreas. There is recurrent epigastric or upper abdominal colicky pain; it radiates towards the left, usually along left costal margin in the direction of the tail of the pancreas. There may be nausea and vomiting along with weight loss, appetite disturbance, and bowel disturbance. Due to fat malabsorption, the stools are bulky and foul smelling, which is called steatorrhea. There may be varying degrees of liver and gall bladder problems accompanying chronic pancreatitis. ERCP is the best diagnostic tool. Acute recurrent pancreatitis is usually a more acute event that is initiated by pathologic activity within the glandular/ductal system. Deep epigastric pain radiates to the back, and it is relieved by leaning forward. There may be an accompanying fever with a history of similar attacks in the past. An inferior MI can mimic this clinical picture, but pain direction, radiation, and EKG would be diagnostic. Acute cholecystitis is more common in women, and it can be excluded by direction and radiation of the pain leaning towards the right hypochondrium. In left ureteric colic, the pain would not present in the epigastrium; instead, it would be located more in the left lumbar region and flank, and it would radiate towards the scrotum on that side.
A 17-year-old male adolescent presents with unexplained neurological symptoms. His liver is enlarged on palpation and he has other symptoms of hepatitis. Blood work reveals depressed ceruloplasmin levels. An ophthalmological examination reveals Kayser-Fleischer rings. The most likely diagnosis is Wilson's disease. What abnormality is the primary cause of the disease? Answer Choices 1 Copper metabolism 2 Selenium deficiency 3 Glycolipid metabolism 4 Glycogen storage 5 Collagen biosynthesis
copper metabolism This young man is suffering from Wilson's disease, a genetic disorder of copper metabolism. It is inherited as an autosomal recessive mutation in the ATP7B genes located on chromosome 13. The protein of the ATP7B gene is a copper-transporting ATPase. The frequency of the heterozygous carriers is relatively high (1/90). The incidence of homozygous recessive affected individuals is about 1/30,000. It affects all ethnic groups and both sexes equally. Neurological symptoms, hepatitis, and Kayser-Fleisher rings, greenish-brown deposits of copper in the corneal endothelial (Descemet membrane) basement membrane near the peripheral cornea where it meets the iris (limbus), are characteristic of Wilson's disease. About 40% of the dietary copper is absorbed in the gastrointestinal tract, and makes its way to the liver bound to albumin. In the liver, it is complexed with ceruloplasmin, a blood protein that carries most of the copper. Ceruloplasmin levels are abnormally low in patients with Wilson's disease, although the disease is not, per se, a mutation in the ceruloplasmin gene. Ceruloplasmin is recycled in the liver by the usual lysosomal degradation pathway, and the unused copper is excreted in bile. When excessive copper is absorbed in the gut, it accumulates in the brain (producing neurological symptoms), the liver (producing hepatitis and hepatomegaly), and the cornea. Penicillamine can be used to treat this disease. It chelates copper and provides some symptomatic relief. Unfortunately, when used appropriately in pregnant women to treat potentially life-threatening Wilson's disease, it is harmful to the fetus and can produce cutis laxa in the newborns of penicillamine-treated patients
A 53-year-old man presents with a 2-day history of jaundice and malaise. His history is significant for mucosal candidiasis, for which the patient is on oral ketoconazole 200 mg daily for the past 3 weeks. He occasionally has headaches, and sometimes takes paracetamol in a daily dose of 1 gram to achieve headache relief. Two weeks before the appearance of symptoms, the patient was treated with flucloxacillin for respiratory tract infection. Except for the appearance of jaundice and malaise, the patient denies the presence of any other symptoms, and the remainder of his personal history is unremarkable. Physical examination reveals a mildly jaundiced patient, 180 centimeters tall, 82 kilograms in weight. His blood pressure is 110/86 mmHg, and the remainder of his general physical examination revealed no abnormalities. Laboratory analyses reveal the presence of hyperbilirubinemia and elevated serum transaminases, normal alkaline phosphatase and γ-Glutamyl transferase levels. Other routine laboratory analyses reveal no abnormalities. Serological testing does not reveal the presence of antibodies against human immuno-deficiency, hepatitis A, C, D and E, or viruses in patient's serum. Also, HBsAg, anti-HBs or anti-HBc antibodies are not present in patient's serum. Anti-LKM-1, antinuclear, anti-thyroid, antimitochondrial, and anti-smooth muscle antibodies are absent. Question What is the most likely cause of liver dysfunction in this patient? Answer Choices 1 Paracetamol-induced hepatitis 2 Primary sclerosing cholangitis 3 Ketoconazole-caused hepatitis 4 Flucloxacillin-induced hepatitis 5 Primary autoimmune hepatitis
ketoconazole induced hepatitis Ketoconazole is an antifungal imidazole used in treating systemic mycoses. The common adverse effect of ketoconazole is hepatotoxicity. The severity of ketoconazole-induced hepatotoxicity is linked to the exposure level of the drug. Ketoconazole-induced hepatotoxicity is probably mediated through a reactive metabolite N- deacetyl ketoconazole (DAK). The latter appears to be the major metabolite, which is a hepatic cytotoxic. Ketoconazole administration results in a significantly dose-dependent increase in serum transaminase activities, as well as cloudy swelling, ballooning degeneration and centrilobular confluent necrosis of the hepatocytes. The histological feature ranges from acute hepatitis to confluent centrilobular, or massive necrosis. Paracetamol hepatotoxicity is dose-related (i.e., signs of liver dysfunction appear only when a high dose of paracetamol, which exceeds liver metabolizing capacity for this drug, is taken). In such a case, toxic intermediary metabolites do accumulate, causing liver damage. It is considered that a daily paracetamol dose of 10 - 15 mg/kg may be considered as a safe, therapeutic dose. Therefore, liver dysfunction, in the presented case, is not the result of paracetamol-induced hepatitis. Primary sclerosing cholangitis is frequently accompanied with elevation of serum alkaline phosphatase, and gamma-glutamyl transferase levels. Since increased serum levels of those enzymes are not noted in the presented case, it is unlikely that the patient suffers from primary sclerosing cholangitis. Flucloxacillin administration causes cholestatic hepatitis (i.e., liver dysfunction in patients with flucloxacillin-induced hepatitis is associated with elevated alkaline phosphatase and gamma-glutamyl transferase levels, as well as with disproportionate rise of conjugated serum bilirubin level). Since hepatitis in the presented patient is not of cholestatic type, it is unlikely that he suffers from flucloxacillin-induced hepatitis. Autoimmune hepatitis is characterized with the appearance of auto-antibodies, including liver kidney microsomal (LKM) antibodies. LKM antibodies in patients with genuine (primary) autoimmune hepatitis are of LKM1 type, not of LKM2 type.
A 45-year-old woman comes to her physician for her regular visit. She has a history of diabetes mellitus for 10 years and is on 10 units of lantus insulin daily at night. She has no complaints at this time and has no other medical history. Family history is unremarkable. She is a nonsmoker and drinks 2-3 beers on the weekends. On examination, she is afebrile, BP 120/80 mmHg, pulse 76/minute, and has no jaundice, pallor, or cyanosis. Lungs are clear, heart sounds are normal, and abdominal exam reveals no hepatosplenomegaly, ascites, tenderness, or mass. Routine labs show a Hb of 13 g%, WBC 7000/uL, platelets 350,000/uL, AST 90 U/L, ALT 102 U/L, AP 36 U/L, bilirubin 1.1 mg/dL, and albumin 4.0 mg/dL. Fasting blood sugar is 116 mg/dL. Further tests show negative HBsAg and anti-HCV. ANA, ESR, and gamma GT are also normal. Ultrasound of the liver is pending. Question What is the most likely diagnosis? Answer Choices 1 Alcoholic liver disease 2 Primary biliary cirrhosis 3 Nonalcoholic steatohepatitis 4 Autoimmune hepatitis 5 Cholestatic hepatitis
nonalcoholic steatohepatitis Explanation Nonalcoholic steatohepatitis (NASH) is a diagnosis of exclusion in patients who have an asymptomatic increase in liver enzymes where no other etiological factor can be determined. It is a definite clinical entity seen most commonly in patients with diabetes, obesity, or hyperlipidemia. In this patient, her alcohol intake is not enough to cause alcoholic hepatitis. Also her ALT is higher than AST, the reverse of that seen in alcohol-related liver disease. Primary biliary cirrhosis and cholestatic hepatitis have an increase in alkaline phosphatase due to cholestasis, which is normal in this patient. Autoimmune hepatitis is usually associated with a high ANA. NASH can also be seen, though uncommonly, with other conditions like extensive abdominal surgery, rapid weight loss or severe starvation, hyperalimentation, and certain drugs like tamoxifen and amiodarone. The pathogenesis of NASH is not fully understood, but there are several theories. Insulin resistance is the most widely accepted one, which leads to hepatic steatosis and steatohepatitis. Other oxidative injuries can also cause inflammation and stimulate steatohepatitis like hepatic iron, intestinal bacteria, deficiency of anti-oxidants, etc. Ultrasound may show fatty liver, and liver biopsy will reveal fatty change with or without fibrosis and cirrhosis. There is no particular treatment for NASH, though better diabetic control, gradual weight loss, and treatment of hyperlipidemia are recommended. Less than 25% progress to cirrhosis of the liver. It has a much better prognosis than alcoholic hepatitis. Primary biliary cirrhosis is a chronic liver disease of middle-aged women of unknown etiology. An immunologic basis has been proposed. Alkaline phosphatase is increased due to cholestasis as well as IgM level. Early symptoms include pruritus and fatigue. Later jaundice, arthralgias, hyperpigmentation, keratconjunctivitis, steatorrhea, osteomalacia, and vitamin deficiencies may occur. Anti-mitochondrial antibodies are positive in more than 90% of patients and are diagnostic. Liver biopsy shows granulomatous inflammation of the bile ducts. Treatment includes replacement of vitamins, ursodeoxycholic acid, cholestyramine for itching, cyclosporine, colchicines, and methotrexate. Studies are ongoing with lamivudine, zidovudine, and benzafibrate. Autoimmune hepatitis is a heterogeneous condition characterized by the presence of autoantibodies, abnormal liver enzymes, absence of other etiological factors, positive ANA, marked increase in serum gamma globulins, and piecemeal necrosis on liver biopsy. Antismooth muscle antibody, soluble liver antigen antibodies, and antibodies to liver/kidney microsomal (LKM) antigens are seen. Steroids and azathioprine are used for treatment.
A 47-year-old man presents with abdominal pain and difficulties breathing. He has a history of alcohol abuse and confirmed cirrhosis of the liver. On examination, you see a malnourished, jaundiced patient with a distended belly. Percussion of the abdomen reveals a huge amount of fluid and wave sign. What is the primary cause of the ascites? Answer Choices 1 Increased albumin production 2 Increased ammonia production 3 Portal hypertension 4 Decreased fluid intake 5 Blockage of the common bile duct
portal hypertension In cirrhosis the fibrotic changes in the liver tissue increase the resistance to blood flow through the organ and this results in increased portal pressure. The lesion is intrahepatic and sinusoidal. Hepatic synthetic failure causes hypoalbuminemia and decreased albumin production, which leads to reduced portal oncotic pressure in cirrhotic patients. Ammonia levels may be elevated, but elevated levels do not cause ascites. Ammonia is a highly toxic metabolic product of the urea cycle. The urea cycle is the only major pathway to remove waste nitrogen, and it usually gets converted to non-toxic urea in the liver. Increased portal pressure, reduced portal oncotic pressure, and elevated aldosterone level with sodium retention cause ascites. Decrease in fluid intake is actually one of the therapeutic strategies for ascites. Blockage of the common bile duct results in jaundice, but it has nothing to do with the development of ascites.
Couvoisier Sign
presence of an enlarged gallbladder which is nontender and accompanied with mild jaundice, the cause is unlikely to be gallstones.
A 47-year-old Hispanic woman presents with a 2-month history of persistent abdominal pains. She becomes uncomfortable after eating, especially after consuming fats, eggs, chocolate, fried foods, fatty foods, and rich desserts. Her pains are primarily located in her right upper abdominal quadrant and often radiate to her right shoulder blade. Physical examination is essentially unremarkable at this time. Her blood pressure is 122/82 mm Hg, she has a pulse of 72 per minute, and a temperature of 98.6°F with respirations 16 per minute. Her abdominal examination reveals no tenderness, no guard, no rebound, and normally active bowel sounds. Her liver and spleen are not palpable. She has no readily palpable abdominal masses. She is not jaundiced. She has no scleral icterus. Question What test should be given in order to confirm the probable diagnosis? Answer Choices 1 Oral cholecystogram 2 X-ray abdomen 3 Ultrasound abdomen 4 CT scan abdomen 5 Biliary scintigraphy 6 MRI abdomen
ultrasound of the abdomen Based on the patient's history, cholelithiasis is suspected. This condition is more common in females of Caucasian or Hispanic descent than in others. The incidence increases with age. Most (80%) of the stones in the US are the cholesterol type. Bilirubin, pigment, and calcium stones constitute the other 20%. Cholelithiasis may be asymptomatic (the presence of gallstones without symptoms), symptomatic (biliary colic), or complicated (e.g. cholecystitis, choledocholithiasis, cholangitis). Ultrasonography is a safe, reliable, and non-invasive test that can be performed at the bedside and is also safe in pregnant women. It is quite sensitive and specific for stones larger than 2 mm. Only about 10-30% gall stones are radio-opaque and hence abdominal X-ray may not be so useful. The oral cholecystogram is the X-ray taken after administering contrast medium to the patient. Although it can identify gall bladder pathology, it requires preparation and is not as convenient as the ultrasound. CT scan is not the first choice in suspected gallstones, although it may prove useful in intrahepatic stones. Biliary scintigraphy is of value in cystic duct obstruction. MRI is also an excellent choice but is expensive, requires sophisticated equipment, and is not used as the first choice investigation in most centers. Symptomatic stones are treated with cholecystectomy. Asymptomatic stones may also require surgery under special circumstances such as large (>2 cm) stones, those with spinal cord injuries affecting the abdomen, and in calcified gall bladder.